Вы находитесь на странице: 1из 54

MyPasTest: MRCS A Online - Jan Exam 2015

02. Anatomy; Thorax(120Qs)


-----------------------------------------------------------------------------------------------------------------
01. A 17-year-old male is stabbed over the left lower ribs whilst travelling home from a night out. He
is alert and haemodynamically stable on admission. A naso gastric tube is inserted and a chest x-
ray shows this to be in the thoracic cavity and a traumatic diaphragmatic injury is thought likely.
What is the best imaging technique to visualise the anatomy of the diaphragm? Select one answer
only.
Abdominal CT YOUR ANSWER Page |
Barium swallow 70
Chest CT
Gastrograffin swallow
MRI CORRECT ANSWER.

Diaphragmatic injuries result from either blunt or penetrating trauma. A traumatic diaphragmatic
rupture is more commonly diagnosed on the left side, perhaps because the liver obliterates the
defect or protects it on the right side. In addition, the appearance of bowel, stomach or a
nasogastric (NG) tube is more easily detected in the left side of the chest. Right diaphragmatic
ruptures are rarely diagnosed in the early post-injury period. The liver often prevents herniation of
other abdominal organs into the chest.

This, however, may not be representative of the true incidence of laterality and autopsy studies
have revealed that left- and right-sided ruptures occur almost equally. Blunt trauma produces large
radial tears measuring 515 cm, most often at the posterolateral aspect of the diaphragm. In
contrast, penetrating trauma usually create only small linear incisions or perforations, which are
less than 2 cm in size and may often take some time, even years, to develop into diaphragmatic
hernias.

If a laceration of the left diaphragm is suspected, a NG tube should be inserted. If the tube appears
in the thoracic cavity on the chest film, the need for special contrast studies can be eliminated.
Minimally invasive endoscopic procedures (thoracoscopy) may be helpful in evaluating the injury to
the diaphragm in indeterminate cases.

Abdominal computed tomography scan is usually not helpful because of its poor visualisation of
the diaphragm. Magnetic resonance imaging is more accurate in visualising the anatomy of the
diaphragm. It is very sensitive and specific and so is the investigation of choice. Surgical repair is
necessary, even for small tears, because the defect will not heal spontaneously.

02. A 32-year-old female complains of a long history of pain and paraesthesiae along the ulnar
border of her left arm and forearm precipitated by placing her upper limb in certain positions. She
reports episodes of dropping items in her left hand recently and also episodes of left arm swelling.
On examination wasting of the small muscles of the left hand is noted. Which of the following is the
most likely diagnosis? Select one answer only.
Carpal tunnel syndrome YOUR ANSWER
Cubital tunnel syndrome
Erbs palsy
Klumpkes paralysis
-------------------------------------------------------------------------------------------------------------------------------------------------------------------------
Dr Mohammed Shamsul Islam Khan, Medical Officer, Clinical Neuro-Surgery, National Institute of Neuro-Sciences and Hospital Sher-E-Bangla
Nagar, Dhaka-1207, Bangladesh. Mobile: +880 1713 455 662, E-mail: drsikhan@gmail.com
MyPasTest: MRCS A Online - Jan Exam 2015
02. Anatomy; Thorax(120Qs)
-----------------------------------------------------------------------------------------------------------------
Thoracic outlet syndrome CORRECT ANSWER.

Cervical rib is present in about 0.5% of the population, of which 60% are symptomatic. Symptoms
due to the presence of a cervical rib depend on the structure it is compressing. Neurological
symptoms are the most common presentation, usually compression of the C8 and T1 nerve roots,
which causes pain and paraesthesiae on the ulnar aspect of the arm and forearm and wasting of the Page |
small muscles of the hand. Vascular changes are seen less often. The arm can become swollen as a
result of venous compression. Compression of the subclavian artery can lead to thrombus 71
formation, emboli, ischaemic changes, and even gangrene.

03. A 65-year-old diabetic with hypertension and a long smoking history is due to undergo a CABG
for triple vessel disease. Which of the following grafts would be likely to have the highest patency
rates at 10 years? Select one answer only.
Left internal mammary YOUR ANSWER
Long saphenous vein
Radial artery
PTFE graft
Cryopreserved allograft vein.

During the first year following a CABG up to 15% of vein grafts occlude. Between 16 years the
graft attrition is 1% to 2% per year and between 610 years it is 4% per year. By 10 years after
bypass surgery, only 60% are patent and only 50% are free of significant stenosis. The internal
thoracic (mammary) artery has a reported patency of up to 90% at 10 years.

The left internal thoracic artery is usually anastomosed to the left anterior descending artery (or
anterior interventricular artery). A number of studies have been carried out to assess the use of
alternative grafts for coronary bypass surgery, including PTFE and cryopreserved allograft veins
(CAVs). The results so far have been disappointing.

04. A 32-year-old man is brought in after a RTA in which he was doing 40 mph and collided with an
on-coming vehicle after he lost concentration momentarily. He was not wearing a seatbelt and a
sternal fracture is seen on imaging. There is concern about a myocardial contusion. What is the
best way to establish this diagnosis? Select one answer only.
ECG YOUR ANSWER
Direct inspection of myocardium
Serial troponin levels and ECG CORRECT ANSWER
Transoesophageal echo
Transthoracic echo.
Blunt cardiac injury, commonly seen in patients with decelerating trauma, can cause valvular
disruption, pericardial effusion, myocardial muscle contusion and cardiac chamber rupture.
Patients with myocardial contusion may complain of chest discomfort/pain and may be
hypotensive. However, cardiogenic shock is rarely seen with myocardial contusion alone.

-------------------------------------------------------------------------------------------------------------------------------------------------------------------------
Dr Mohammed Shamsul Islam Khan, Medical Officer, Clinical Neuro-Surgery, National Institute of Neuro-Sciences and Hospital Sher-E-Bangla
Nagar, Dhaka-1207, Bangladesh. Mobile: +880 1713 455 662, E-mail: drsikhan@gmail.com
MyPasTest: MRCS A Online - Jan Exam 2015
02. Anatomy; Thorax(120Qs)
-----------------------------------------------------------------------------------------------------------------

A two-dimensional echocardiography may reveal an abnormal ventricular wall motion. The


electrocardiographic changes are variable: They include multiple premature ventricular
contractions, unexplained sinus tachycardia, atrial fibrillation, bundle branch block (usually right),
non-specific ST and T wave changes and features suggestive of frank myocardial infarction.

In current practice, the widespread availability of cardiospecific troponin I and troponin T assays Page |
has made it easier to detect myocardial injury, especially in patients with no major signs of cardiac 72
injury. Troponin I and troponin T have also facilitated the stratification of patients at risk for life
threatening complications.

Serial measurements of troponin I or T in combination with ECG have shown to be sufficient for
identifying the vast majority of patients at risk, thus avoiding extensive diagnostic screening.
Moreover, patients with normal troponin I or T concentrations and ECG may be safely discharged
after a period of observation.

Ref: http://www.ncbi.nlm.nih.gov/pmc/articles/PMC1767619/

05. A 64-year-old man is admitted with a tearing chest pain radiating through to his back. He is
haemodynamically stable and a CT angiogram shows a type A thoracic aneurysm with dissection.
Which of the following vessels normally arise from the aortic arch? Select one answer only.
Left subclavian artery YOUR ANSWER (Correct)
Left vertebral artery
Right internal mammary artery
Right common carotid artery
Right subclavian artery.

The arch of the aorta commences from the manubriosternal joint and passes backwards over the
left bronchus to reach the body of T4 vertebra just to the left of the midline. The arch is crossed on
its left side by the phrenic and vagus nerves as they pass downwards in front of and behind the
lung root, respectively. The left vertebral artery and the right internal mammary artery come off the
left and right subclavian arteries, respectively.

06. Following excision of a left cervical rib a 28-year-old female is found to have a milky, white fluid
in her drain. What structure is most likely to have been damaged? Select one answer only.
Cisterna chyli YOUR ANSWER
Hemiazygos vein
Left subclavican artery
Left subclavican vein
Thoracic duct CORRECT ANSWER.

-------------------------------------------------------------------------------------------------------------------------------------------------------------------------
Dr Mohammed Shamsul Islam Khan, Medical Officer, Clinical Neuro-Surgery, National Institute of Neuro-Sciences and Hospital Sher-E-Bangla
Nagar, Dhaka-1207, Bangladesh. Mobile: +880 1713 455 662, E-mail: drsikhan@gmail.com
MyPasTest: MRCS A Online - Jan Exam 2015
02. Anatomy; Thorax(120Qs)
-----------------------------------------------------------------------------------------------------------------
The thoracic duct leaves the cysterna chyli at the level of L1L2, ascends into the thorax to the right
of the descending thoracic aorta, crosses the midline gradually to reach the left border of the
oesophagus (plane of Louis, T4), where it continues to run upwards, reaching the root of the neck.

It then curves behind the carotid sheath and arches over the left subclavian artery to drain into the
left brachiocephalic vein, although it can also drain into the other adjacent major veins. It carries
lymph from the lower limbs, abdominal and pelvic regions, left thorax, left head and neck plus the Page |
left arm. 73
07. A 27-year-old male is rushed into A&E after being involved in a RTA. From the history he was a
front seat passenger in a stolen car that crashed into a wall at approximately 70mph. He is
complaining of generalised pain and on examination prominent marks are seen corresponding to
his seatbelt. His trauma series chest x-ray shows an obliterated aortic knuckle and there is concern
about blunt aortic rupture. Which of the following imaging modalities is the gold standard in
diagnosing this? Select one answer only.
Aortography YOUR ANSWER (Correct)
Contrast- enhanced CT of the chest
MRI
Transoesophageal echocardiography
Transthoracic echocardiography.

Traumatic aortic disruption, a time-sensitive injury, is a common cause of sudden death after an
automobile collision or a fall from great height. A complete tear through the tunica intima, media
and adventitia usually leads to rapid exsanguination and death. In aortic rupture survivors,
immediate death is prevented due to the vascular continuity maintained by a pseudoaneurysm
within an intact adventitial layer or a mediastinal haematoma.

A large mediastinal haematoma may shift the trachea to the right. This condition has a variable
course ranging from a relatively clinically silent period due to the contained rupture
(pseudoaneurysm), to rupture of the pseudoaneurysm, exsanguination and death. Radiographic
findings may include a widened mediastinum, obliteration of the aortic knuckle, deviation of the
trachea to the right, obliteration of the space between the pulmonary artery and the aorta
(obscuration of AP (aorto-pulmonary) window), depression of the left main stem bronchus,
deviation of the oesophagus (nasogastric tube) and fractures of the first or second rib or scapula.

False-positive and false-negative findings occur with each radiographic sign and, rarely (12%), no
mediastinal or initial chest X-ray abnormality is present in patients with great vessel injury.
Although transoesophageal echocardiography is a useful, the less invasive diagnostic tool,
aortography is the gold standard in the diagnosis of blunt aortic rupture. Helical contrast-enhanced
computed tomography (CT) of the chest is also an accurate screening method for patients with
suspected blunt aortic injury. However, a patient who is haemodynamically abnormal should not be
placed in a CT scanner.

In stable patients, if enhanced helical CT of the chest is negative for mediastinal haematoma and
aortic rupture, no further diagnostic imaging is necessary. If it is positive for blunt aortic rupture,
the extent of the injury can best be ascertained by aortography.
-------------------------------------------------------------------------------------------------------------------------------------------------------------------------
Dr Mohammed Shamsul Islam Khan, Medical Officer, Clinical Neuro-Surgery, National Institute of Neuro-Sciences and Hospital Sher-E-Bangla
Nagar, Dhaka-1207, Bangladesh. Mobile: +880 1713 455 662, E-mail: drsikhan@gmail.com
MyPasTest: MRCS A Online - Jan Exam 2015
02. Anatomy; Thorax(120Qs)
-----------------------------------------------------------------------------------------------------------------

08. In a 27-year-old restrained passenger from a high speed collision with a tree, you suspect a
deceleration injury to the aorta. Which feature on chest X-ray would most reliably support your
suspicion? Single best answer question choose ONE true option only.
Obliteration of the aortic knuckle YOUR ANSWER
Obscuration of the aorto-pulmonary window Page |
Presence of a pleural cap 74
Depression of the left main stem bronchus
Widened mediastinum CORRECT ANSWER.

All of the above can be associated with a traumatic aortic injury. However, they are non-specific
signs with high false positive and false negative rates. Occasionally, no chest X-ray changes are
visible following traumatic aortic injury. In a trauma supine CXR, a widened mediastinum is the
most consistent feature of aortic disruption. About 3% of these patients would prove to have aortic
injury on contrast CT or aortic angiogram.

09. You are called to the Trauma Room of the Emergency Department and find a 34-year-old
motorcyclist with a blunt chest injury. On examination you notice reduced breath sounds and a dull
percussion note over the right hemithorax. Trauma care is initiated and includes the insertion of an
intercostal drain which quickly yields 1600mls of blood. His blood pressure is recorded as
100/65mmHg. What is the most appropriate management of this injury?
Continued non-surgical management YOUR ANSWER
Emergency thoracoscopy
Emergency thoracotomy in the Emergency Department
Emergency thoracotomy in the Operating Theatre CORRECT ANSWER
Massive blood transfusion.

This patient has a massive haemothorax, which is defined as the rapid accumulation of more than
1500mL (or one third) of a patients blood volume in the chest cavity. ATLS guidelines recommend
that emergency thoracotomy is performed by a thoracic surgeon in an operating theatre
environment in such cases. Blood transfusion may be required, but massive transfusion may not
be necessary if surgical methods of haemostasis are rapidly delivered.

Emergency room thoracotomy should only be performed in the context of:


i) penetrating chest injury plus cardiac arrest (with previously witnessed cardiac activity) or
unresponsive hypotension (<70mmHg SBP).
ii) blunt chest injury with massive haemothorax and unresponsive hypotension (<70mmHg).

10. During pericardiectomy, sudden bleeding was noticed due to accidental injury to a major
vascular structure in the pericardium. The surgeon inserted his left index finger through the
transverse pericardial sinus, pulled forward on the two large vessels lying ventral to his finger and
compressed these vessels with his thumb to control bleeding. Which vessels were these? Single
best answer question choose ONE true option only.
-------------------------------------------------------------------------------------------------------------------------------------------------------------------------
Dr Mohammed Shamsul Islam Khan, Medical Officer, Clinical Neuro-Surgery, National Institute of Neuro-Sciences and Hospital Sher-E-Bangla
Nagar, Dhaka-1207, Bangladesh. Mobile: +880 1713 455 662, E-mail: drsikhan@gmail.com
MyPasTest: MRCS A Online - Jan Exam 2015
02. Anatomy; Thorax(120Qs)
-----------------------------------------------------------------------------------------------------------------
Pulmonary trunk and brachiocephalic trunk YOUR ANSWER
Pulmonary trunk and aorta CORRECT ANSWER
Pulmonary trunk and superior vena cava
Superior vena cava and aorta
Page |
Superior vena cava and right pulmonary artery.
75
The transverse pericardial sinus is an area of the pericardial cavity located behind the aorta and
pulmonary trunk and anterior to the superior vena cava. Therefore, the two large vessels lying
ventral to his finger are the pulmonary trunk and aorta; the large vessel lying dorsal to his finger is
the superior vena cava.

11. A 58-year-old diabetic male presents with chest pain radiating to the back and an initial chest X
ray shows a widened mediastinum. Further imaging demonstrates a thoracic aortic aneurysm and
the decision is made for surgical management. What type of incision allows the best access to the
thoracic aorta? Select one answer only.
Median sternotomy YOUR ANSWER
Rooftop incision
Thoracoabdominal
Thoracoscopic
Thoracotomy CORRECT ANSWER.

Thoracic aortic aneurysms are far less common than abdominal aneurysms. Previous dissection
may lead to weakness of the aortic wall, and thus predispose to aneurysm formation. There is no
common association with neurofibromatosis. They are just as likely to rupture as abdominal
aneurysms; according to the law of LaPlace the wall tension is proportional to the radius at a given
pressure. Access to the thoracic aorta is best afforded by a thoracotomy rather than a median
sternotomy, as it is a posterior mediastinal structure.

12. A 38-year-old man is diagnosed with a sliding hiatus hernia. Which pair of structures travel
through the same diaphragmatic aperture as the hernia? Select one answer only.
Azygous vein and inferior vena cava YOUR ANSWER
Azygous vein and right phrenic nerve
Hemiazygous vein and oesophagus
Vagus nerve and azygous vein
Vagus nerve and oesophagus CORRECT ANSWER.

The sliding hernia occurs as a result of the stomach and oesophagus moving cranially through the
oesophageal opening of the diaphragm into the thoracic cavity.

-------------------------------------------------------------------------------------------------------------------------------------------------------------------------
Dr Mohammed Shamsul Islam Khan, Medical Officer, Clinical Neuro-Surgery, National Institute of Neuro-Sciences and Hospital Sher-E-Bangla
Nagar, Dhaka-1207, Bangladesh. Mobile: +880 1713 455 662, E-mail: drsikhan@gmail.com
MyPasTest: MRCS A Online - Jan Exam 2015
02. Anatomy; Thorax(120Qs)
-----------------------------------------------------------------------------------------------------------------
Three sets of structures pierce the diaphragm at three vertebral levels:

1. T8 = Right phrenic nerve and IVC; these pierce the diaphragm through the central tendon
(caval opening)
2. T10 = Vagus nerve and Oesophagus; these pierce the diaphragm through its right crus
(oesophageal opening)
3. T12 = Azygous vein, thoracic duct and abdominal aorta; these pass posterior to the Page |
diaphragm through the aortic opening. 76
13. A 20-year-old man presents to A&E with a traumatic pneumothorax. He is obviously tachypnoeic
but maintains adequate oxygen saturation. Which of the following is least likely to contribute to his
respiratory effort? Select one answer only.
Contraction of the intercostal muscles YOUR ANSWER
Descent of the hemidiaphragms
An increase in the vertical dimension of the chest
The long thoracic nerve of Bell (supplying the serratus anterior)
Upward/forward movement of the first rib CORRECT ANSWER.

In cases of impaired ventilation, tachypnoea can compensate the impaired gas exchange to an
extent until fatigue of the respiratory muscles occurs. A number of factors are involved in
respiration:
- An increase in the vertical dimension of the chest on inspiration
- An upwards and outwards movement of the ribs
- A rise and fall of the hemi diaphragms.

The serratus anterior (supplied by the long thoracic nerve) is also involved in respiration.

Note that the first rib does not move during respiration.

14. A 76-year-old male is undergoing an oesophagectomy for adenocarcinoma of the distal


oesophagus thought to be secondary to long standing GORD. Which part of the diaphragm does
the oesophagus pass through? Select one answer only.
Central tendon YOUR ANSWER
Left crus
Median arcuate ligament
Peripheral muscular part
Right crus CORRECT ANSWER.

It is important to know what structures pass through the diaphragm, at what vertebral levels, and
also which specific part of the diaphragm. The aortic opening lies behind the diaphragm at T12 at
vertebral level T12. It contains the aorta, thoracic duct, and often the azygos vein.

The oesophageal hiatus is at vertebral level T10 behind the seventh costal cartilage, and is formed
-------------------------------------------------------------------------------------------------------------------------------------------------------------------------
Dr Mohammed Shamsul Islam Khan, Medical Officer, Clinical Neuro-Surgery, National Institute of Neuro-Sciences and Hospital Sher-E-Bangla
Nagar, Dhaka-1207, Bangladesh. Mobile: +880 1713 455 662, E-mail: drsikhan@gmail.com
MyPasTest: MRCS A Online - Jan Exam 2015
02. Anatomy; Thorax(120Qs)
-----------------------------------------------------------------------------------------------------------------
by the medial fibres of the right crus of the diaphragm as they arch across to the left side of the
midline. They form a tubular envelope around the oesophagus with an elliptical opening.

In addition to the oesophagus, it transmits the vagus nerve, left gastric artery and vein.

The inferior vena cava passes through at T8 in the central tendon. The right phrenic nerve passes
with it. Page |
77
The sympathetic trunk passes posterior to the medial arcuate ligament lying on psoas major. Each
half of the diaphragm is supplied by its own phrenic nerve (C3, C4, C5) which is both motor and
sensory. The intercostal nerves send some proprioceptive fibres to the periphery of the diaphragm.
The greater, lesser and least splanchnic nerves pierce each crus.

15. A 4-year-old boy is rushed into A&E by his mum who is concerned he has aspirated on one of
his lego bricks he likes to put into his mouth. On examination, the child appears well with sats of
98%. A chest X-ray shows a foreign body impacted in the right main bronchus. Which of the
following is the most appropriate management? Select one answer only.
Conservative management YOUR ANSWER
CT guided removal
Fibre-optic bronchoscopy to attempt to remove it
Rigid bronchoscopy to attempt to remove it CORRECT ANSWER
Thoracoscopic removal.

Inhaled or aspirated foreign bodies are more commonly seen in children than adults. A history of
foreign body aspiration must be taken seriously, as the consequences of a retained foreign body in
the lung may be significant. Following a careful history and examination, posteroanterior and lateral
chest radiography may help to localise the foreign object. However, some objects may be
radiolucent and may not be seen on the chest radiograph.

Further investigation is imperative, the next step being either fibre-optic or preferably rigid
bronchoscopy. Rigid bronchoscopy is preferred for recovery of foreign body aspiration because it
allows protection of the airway and controlling the foreign body during recovery. The anatomy of
the bronchial tree, in particular a less angulated right main bronchus, makes this the commonest
site of impaction.

16. A 62-year-old male with alcoholic liver disease is deteriorating in ICU following insertion of a
Sengstaken-Blakemore tube for bleeding oesophageal varices. His chest X-ray shows a
pneumomediastinum and he is thought to have perforated his oesophagus. Which of the following
structures which run adjacent to the oesophagus makes an indentation on the left lung, but not the
right lung? Select one answer only.
Azygos vein YOUR ANSWER
Oesophagus
Phrenic nerve
Superior vena cava
-------------------------------------------------------------------------------------------------------------------------------------------------------------------------
Dr Mohammed Shamsul Islam Khan, Medical Officer, Clinical Neuro-Surgery, National Institute of Neuro-Sciences and Hospital Sher-E-Bangla
Nagar, Dhaka-1207, Bangladesh. Mobile: +880 1713 455 662, E-mail: drsikhan@gmail.com
MyPasTest: MRCS A Online - Jan Exam 2015
02. Anatomy; Thorax(120Qs)
-----------------------------------------------------------------------------------------------------------------
Thoracic duct CORRECT ANSWER.

Impressions on the mediastinal surface of the right lung include the trachea, vagus, superior vena
cava, right atrium and subclavian artery. The oesophagus grooves the left lung above the arch of
the aorta and below the hilum. There is also a vertical groove for the oesophagus on the right lung.
Page |
17. During a coronary angiography a tight stenosis is noted at the proximal aspect of the left
coronary artery. Which of the following is the most important branch of the left coronary artery? 78
Select one answer only.
Anterior interventricular YOUR ANSWER (Correct)
Atrioventricular nodal
Circumflex
Marginal
Posterior interventricular.

The left coronary artery divides into the left anterior interventricular artery also known as the left
anterior descending artery (LAD) and circumflex artery. Its most important branch is the anterior
interventricular artery which supplies the anterior aspect of both ventricles and passes around the
apex of the heart to anastomose with the posterior interventricular artery. The right coronary artery
gives off a posterior interventricular branch. The right coronary artery supplies the AV node in 80%
of cases.

18. A 25-year-old female presents with pain in the right hand which is associated with a tingling
sensation and weakness. After further investigation she is diagnosed with thoracic outlet
syndrome. The affected neurovascular bundle runs between scalenus anterior and medius. Where
do these muscles insert?
Body of the sternum YOUR ANSWER
Clavicle
First rib CORRECT ANSWER
Manubrium
Second rib.

Thoracic outlet syndrome is caused by compression of the neurovascular bundle that passes
between the anterior and middle scalene muscles. The brachial plexus and the subclavian artery
and (rarely) the subclavian vein can be affected, leading to a variety of upper limb, neck and upper
back symptoms. Cervical ribs, Pancoasts tumour and preganancy can all cause the condition, but
it is usually secondary to trauma or repetitive strain injuries.

The scalene muscle is composed of three pairs of constituent muscles, namely scalenus anterior
(originating at anterior tubercles of the transverse processes of C3-6 vertebrae), scalenus medius
(originating from the posterior tubercles of C2-6 vertebrae) and scalenus posterior (originating from

-------------------------------------------------------------------------------------------------------------------------------------------------------------------------
Dr Mohammed Shamsul Islam Khan, Medical Officer, Clinical Neuro-Surgery, National Institute of Neuro-Sciences and Hospital Sher-E-Bangla
Nagar, Dhaka-1207, Bangladesh. Mobile: +880 1713 455 662, E-mail: drsikhan@gmail.com
MyPasTest: MRCS A Online - Jan Exam 2015
02. Anatomy; Thorax(120Qs)
-----------------------------------------------------------------------------------------------------------------
the posterior tubercles of the transverse processes of C4-6). The anterior and medius muscles
insert into the first rib, the posterior component inserts into the second.

19. A 62-year-old ex-smoker has been diagnosed with non-small cell lung carcinoma by
endobronchial biopsy. At standard mediastinoscopy, what is the least likely lymph node station to
be sampled? Single best answer question choose ONE true option only.
Page |
Paratracheal nodes YOUR ANSWER
79
Subcarinal nodes
Tracheobronchial nodes
Aortopulmonary nodes CORRECT ANSWER
Pretracheal nodes .

During mediastinoscopy, the pretracheal fascia is incised and blunt dissection creates a tunnel
inferiorly. The linear passage of the mediastinoscope along this tunnel allows visualisation of the
nodes lying to the front (pretracheal) and sides (paratracheal) of the trachea. The subcarinal and
tracheobronchial nodes can also be reached at the distal end of this tunnel. The aortopulmonary
nodes, however, cannot easily be reached as the aorta is in the way of the advancing finger or
mediastinoscope.

20. Following a difficult tracheostomy, the surgeon is concerned about the nerve supply to the
trachea being damaged. Which of the following nerves supply the trachea? Select one answer only.
Phrenic nerve YOUR ANSWER
Glossopharyngeal nerve
Recurrent laryngeal nerve CORRECT ANSWER
Superior laryngeal nerve
External laryngeal nerve.

The trachea attaches to the larynx via the cricotracheal membrane at about the level of the sixth
cervical vertebrae, and descends through the neck and superior mediastinum to terminate at about
the level of the disc between the fourth and fifth thoracic vertebrae, although this bifurcation
descends as the trachea is stretched during inspiration.

The tracheal rings consist of hyaline cartilage. It is supplied by the recurrent laryngeal nerve which
is sensory below the level of the vocal cords and motor to all muscles of the larynx with the
exception of the cricothyroid, which is supplied by superior laryneal nerve.

21. A 76-year-old male who is a long term smoker presents with a headache and facial swelling. On
further questioning he reports episodes of haemoptysis and 11/2 stone in weight loss. Examination
reveals a plethoric face and neck, and distended veins on his upper chest and neck. A CT scan
reveals a bronchial tumour causing SVC obstruction. At what level does the SVC enter the right
atrium? Select one answer only.
At the level the azygos vein drains into it YOUR ANSWER

-------------------------------------------------------------------------------------------------------------------------------------------------------------------------
Dr Mohammed Shamsul Islam Khan, Medical Officer, Clinical Neuro-Surgery, National Institute of Neuro-Sciences and Hospital Sher-E-Bangla
Nagar, Dhaka-1207, Bangladesh. Mobile: +880 1713 455 662, E-mail: drsikhan@gmail.com
MyPasTest: MRCS A Online - Jan Exam 2015
02. Anatomy; Thorax(120Qs)
-----------------------------------------------------------------------------------------------------------------
Behind the first costal cartilage
Behind the third costal cartilage CORRECT ANSWER
Level of sternal angle
Vertebral level T8.
Page |
The SVC drains all the structures above the diaphragm except the heart and lungs. It also receives 80
the azygos vein, which drains the lumbar and subcostal regions. The SVC is formed behind the first
costal cartilage by the union of the right and left brachiocephalic veins. It ends behind the third
costal cartilage as it enters the right atrium. The SVC has no valves. The thoracic duct drains into
the left brachiocephalic vein (or sometimes into the subclavian or internal jugular vein).

22. A 17-year-old girl is brought to A&E following a horse riding accident. She is very distressed
and in a lot of pain. From the history the horse fell backwards and landed on the left side of her
chest. On examination there is extensive bruising over the left chest and it can be seen to move in
when she inspires. Auscultation reveals bilateral air entry and her trachea is central. What is the
most likely diagnosis? Select one answer only.
Chylothorax YOUR ANSWER
Flail chest CORRECT ANSWER
Haemothorax
Pneumothorax
Tension pneumothorax.

A flail chest occurs when a segment of the thoracic wall does not have bony continuity with the rest
of the thoracic cage. Flail chest classically results from blunt trauma, as in high-speed road-traffic
accidents. A transfer of significant kinetic energy to the rib cage can cause fracture of the ribs in
multiple areas leading to a segment of the thoracic wall to float independently of the rest of the
chest wall. A flail chest can arise when two or more ribs are fractured in two or more places.

The presence of a flail chest segment results in severe disruption of normal chest wall movement
leading to paradoxical breathing (when the injured segment of the thoracic cage moves in an
opposite direction to the rest of the chest wall). Initial management of flail chest includes adequate
ventilation, administration of humidified oxygen, adequate analgesia and fluid resuscitation.

However, in the absence of systemic hypotension (from other associated causes), intravenous fluid
resuscitation should be carefully monitored to prevent over hydration. The injured lung in a flail
chest is very sensitive to both under-resuscitation and fluid overload. The definitive management of
flail chest is to re-expand the affected (contused or collapsed) lung, ensure adequate oxygenation,
provide sufficient pain relief and judicious fluid resuscitation.

Since prevention of hypoxia is of paramount importance some patients will benefit from a short
period of intubation and ventilation, the timing guided by the respiratory rate, arterial oxygen
tension and other vital respiratory parameters. Surgical stabilisation of the chest is an option but is
rarely necessary in the management of flail chest.

-------------------------------------------------------------------------------------------------------------------------------------------------------------------------
Dr Mohammed Shamsul Islam Khan, Medical Officer, Clinical Neuro-Surgery, National Institute of Neuro-Sciences and Hospital Sher-E-Bangla
Nagar, Dhaka-1207, Bangladesh. Mobile: +880 1713 455 662, E-mail: drsikhan@gmail.com
MyPasTest: MRCS A Online - Jan Exam 2015
02. Anatomy; Thorax(120Qs)
-----------------------------------------------------------------------------------------------------------------
23. You have performed a liver biopsy, and shortly after the procedure the patient develops pain on
the tip of his right shoulder. Which nerve is most likely to be responsible for his pain? Single best
answer - choose ONE true option only.
Right phrenic nerve YOUR ANSWER (Correct)
Axillary nerve Page |
Right vagus 81
Right sympatheticus
Intercostobrachial nerve.
The phrenic nerve on both sides originates from the ventral rami of the third to fifth cervical
nerves. It passes inferiorly down the neck to the lateral border of the scalenus anterior, then it
passes medially across the border of scalenus anterior parallel to the internal jugular vein that lies
inferomedially.
The right phrenic nerve pierces the diaphragm in its tendinous portion just slightly lateral to the
inferior vena caval foramen. It then forms three branches on the inferior surface of the diaphragm:
anterior, lateral and posterior. These ramify out in a radial manner from the point of perforation to
supply all but the periphery of the muscle.
24. You are required to insert a chest drain in a patient with penetrating trauma. Which anatomical
landmark is least useful to you? Single best answer question choose ONE true option only.
Anterior border of latissimus dorsi YOUR ANSWER
The mid-clavicular line CORRECT ANSWER
The sixth rib
The mid-axillary line
Inferolateral border of pectoralis major.

The safe triangle for the insertion of an intercostal drain is bounded anteriorly by the inferolateral
border of pectoralis major, posteriorly by the anterior border of latissimus dorsi and inferiorly by
the axial plane at the level of the nipple. In practice, however, one must note that the position of the
nipple is highly variable and so the drain is best sited within the 5th intercostal space (i.e.
immediately above the 6th rib as located by palpation). The mid-axillary line may be used within this
triangle to help guide placement of the incision. The mid-clavicular line should not be used for
placement of an intercostal tube drain.

25. A pre-operative patient is on -blockers. Where in the thoracic cage are the beta-1
adrenoceptors concentrated? Single best answer question choose ONE true option only.
Cardiac ventricles YOUR ANSWER (Correct)
Pulmonary trunk
Aortic arch
Lungs
Carotid sinus.

-------------------------------------------------------------------------------------------------------------------------------------------------------------------------
Dr Mohammed Shamsul Islam Khan, Medical Officer, Clinical Neuro-Surgery, National Institute of Neuro-Sciences and Hospital Sher-E-Bangla
Nagar, Dhaka-1207, Bangladesh. Mobile: +880 1713 455 662, E-mail: drsikhan@gmail.com
MyPasTest: MRCS A Online - Jan Exam 2015
02. Anatomy; Thorax(120Qs)
-----------------------------------------------------------------------------------------------------------------
The beta-1 adrenoceptors are located within the ventricles of the heart. The atria contain
cholinergic receptors. The aortic arch contains baroreceptors. The Lungs contain beta-2
adrenoceptors. The carotid sinus contains baroreceptors and is in the neck, not the thorax.

26. A 51-year-old female is rushed in following a road traffic accident. The patient was
haemodynamically stable and a right sided haemothorax was noted. A CT scan showed a contained
haematoma of the azygos vein. At what level does the azygos vein enter the superior vena cava? Page |
Select one answer only. 82
T2 YOUR ANSWER
T4 CORRECT ANSWER
T6
T8
T10.

The azygos vein receives blood from the posterior intercostal veins and segmental vains of the
abdomen, It is located on the right, and on the left a variable pattern exists, which mostly involves
an accessory hemiazygos vein (superiorly) and azygos vain (inferiorly).

The azygos vein is formed at the level of the right renal vein (either as a posterior tributary of the
IVC, or as a confluence of the right ascending lumbar vein and right subcostal vein). It passes
through the diaphragm via the aortic opening at T12 and ascends on the right side of the vertebral
bodies posterior to the oesophagus. It terminates by arching over the hilum of the right lung to
enter the SVC at T4 level.

The azygos vein does not extend higher than T4, and therefore the 2nd, 3rd and 4th right intercostal
veins drain into the right superior intercostal vein which itself drains into the azygos vein. Other
tributaries draining into it include the lower 8 right posterior intercostal veins, bronchial and
oesophageal veins and the 2 hemiazygos veins.

27. Following a very difficult right thoracic outlet syndrome decompression in a 35-year-old female
there is some respiratory difficulty and there is concern the right phrenic nerve may be injured.
Which of these descriptions most accurately describes the muscle relations of the phrenic nerve?
Select one answer only.
Anterior to scalenus anterior YOUR ANSWER (Correct)
Anterior to scalenusmedius
Anterior to trapezius
Posterior to scalenus anterior
Posterior to scalenusmedius.

The phrenic nerve arises from C3, C4 and C5 deep to the scalenus anterior and medius muscles,
and runs on scalenus anterior, over the anterior part of the dome of the pleura, to enter the
mediastinum posterior to the subclavian vein. Here the right phrenic nerve spirals forward to lie on

-------------------------------------------------------------------------------------------------------------------------------------------------------------------------
Dr Mohammed Shamsul Islam Khan, Medical Officer, Clinical Neuro-Surgery, National Institute of Neuro-Sciences and Hospital Sher-E-Bangla
Nagar, Dhaka-1207, Bangladesh. Mobile: +880 1713 455 662, E-mail: drsikhan@gmail.com
MyPasTest: MRCS A Online - Jan Exam 2015
02. Anatomy; Thorax(120Qs)
-----------------------------------------------------------------------------------------------------------------
the SVC, right atrium and inferior vena cava (IVC), and traverses the diaphragm via the caval orifice.
The vagus nerve gives off the recurrent laryngeal nerve.

28. A patient with a history of stable angina undergoes a coronary angiography. This shows triple
vessel disease. What is the usual treatment for such a patient? Select one answer only.
Angioplasty YOUR ANSWER Page |
Angioplasty and stent insertion 83
Coronary artery bypass grafting CORRECT ANSWER
Medical treatment
Surveillance.

According to the Coronary Artery Surgery Study the patient groups that derive particular benefit
from coronary artery bypass grafting (CABG) are those with triple vessel disease, and those with
>50% left main stem stenosis. Those with single or double vessel disease are usually more
amenable to percutaneous intervention. Post-myocardial infarction, unstable angina is a primary
indication for urgent CABG. Valvular disease with concomitant coronary artery disease is usually
treated operatively.

29. During a difficult oesophagectomy in a 55-year-old man there is concern the thoracic duct may
have been injured in the mediastinum. Which of the following descriptions best describes the route
of the thoracic duct within the mediastinum in relation to the oesophagus? Select one answer only.
It passes anterior and to the left of the oesophagus at T5 level YOUR ANSWER
It passes anterior and to the right of the thoracic duct at T5 level
It passes posterior and to the left of the oesophagus at T5 level CORRECT ANSWER
It passes posterior and to the left of the oesophagus at T10 level
It passes posterior and to the right of the thoracic duct at T5 level.

The thoracic duct begins below the diaphragm as the cysternachyli and then ascends through the
aortic opening in the diaphragm, to the right of the descending aorta. It passes behind the
oesophagus and then to the left of the oesophagus at the level of T5. It then runs upwards on the
left side of the oesophagus into the neck. Here it crosses the subclavian artery to enter the left
brachiocephalic vein.

At the root of the neck, the thoracic duct receives the left jugular, subclavian and
bronchomediastinal lymph trunks, although they may occasionally drain directly into the adjacent
large vessels. The thoracic duct therefore conveys all the lymph from the lower limbs, pelvic cavity,
abdominal cavity, left side of the thorax, head and neck and the left arm.

30. In the clinical examination of the chest, accurate knowledge of the surface markings of the
lungs is essential. Which of the following corresponds to the clinical situation? Single best answer
question choose ONE true option only.

-------------------------------------------------------------------------------------------------------------------------------------------------------------------------
Dr Mohammed Shamsul Islam Khan, Medical Officer, Clinical Neuro-Surgery, National Institute of Neuro-Sciences and Hospital Sher-E-Bangla
Nagar, Dhaka-1207, Bangladesh. Mobile: +880 1713 455 662, E-mail: drsikhan@gmail.com
MyPasTest: MRCS A Online - Jan Exam 2015
02. Anatomy; Thorax(120Qs)
-----------------------------------------------------------------------------------------------------------------
The apex of the lung corresponds precisely to the upper border of the medial third of the clavicle
YOUR ANSWER
The oblique fissure of the lung corresponds to the medial border of the scapula when the arm is fully
abducted CORRECT ANSWER
The transverse (horizontal) fissure of the right lung corresponds to the right fifth intercostal space Page |
The lower border of the lung on each side corresponds to the tenth rib in the mid-axillary line 84
The lower border of the lung reaches the twelfth rib posteriorly.

The apex of the lung extends about 4 cm above the medial one-third of the clavicle. The oblique
fissure does indeed correspond closely to the medial border of the scapula when the arm is fully
abducted. The transverse fissure of the right lung corresponds to the level of the fourth rib. The
lower border of the lung on each side corresponds to the eighth rib in the mid-axillary line and the
tenth rib posteriorly.

31. A 19-year-old male is brought into A & E following a stabbing. On examination a wound is seen
in the left supraclavicular fossa, but he is otherwise haemodynamically stable. A chest X-ray shows
the left hemidiaphragm is elevated and there is concern that the phrenic nerve may be affected.
Which of the following descriptions best describe the route by which the phrenic nerve enters the
chest? Select one answer only.
Anterior to subclavian artery and posterior to the subclavian vein. Medial to the lower border of
scalenus anterior YOUR ANSWER (Correct)
Anterior to the subclavian artery and posterior to the subclavian vein. Lateral to the lower border of
scalenus anterior
Anterior to the subclavian vein and artery. Medial to the lower border of scalenus anterior
Posterior to subclavian artery and vein. Medial to the lower border of scalenus anterior
Posterior to subclavian vein and artery. Lateral to the lower border of scalenus anterior.

The autonomic fibres in the phrenic nerve are sympathetic and pass from the superior (C1C4) and
middle (C5/6) sympathetic cervical ganglia as grey rami into the C3C5 roots of the phrenic nerve,
and innervate blood vessels in the diaphragm. The nerve lies on the fibrous pericardium and is
sensory to the mediastinal and diaphragmatic pleura, and also to the diaphragmatic peritoneum.
The phrenic nerve enters the chest by descending from the medial lower border of the scalenus
anterior muscle between the subclavian vein anteriorly and artery posteriorly.

32. You are called to the emergency room to see a fifty-six year old female in acute respiratory
distress. The A&E registrar informs you that the patient presented with severe sepsis and so a
central line has been introduced to optimise emergency management. However, they remain
concerned by the patients tachypnoea, cyanosis and falling oxygen saturations. On examination
you notice that there is hyperresonance and absent breath sounds in the right hemithorax. What is
the most likely diagnosis?
Flail chest YOUR ANSWER
Massive haemothorax
-------------------------------------------------------------------------------------------------------------------------------------------------------------------------
Dr Mohammed Shamsul Islam Khan, Medical Officer, Clinical Neuro-Surgery, National Institute of Neuro-Sciences and Hospital Sher-E-Bangla
Nagar, Dhaka-1207, Bangladesh. Mobile: +880 1713 455 662, E-mail: drsikhan@gmail.com
MyPasTest: MRCS A Online - Jan Exam 2015
02. Anatomy; Thorax(120Qs)
-----------------------------------------------------------------------------------------------------------------
Open pneumothorax
Pleural effusion
Tension pneumothorax CORRECT ANSWER.

The combination of hyperresonance and reduced air entry should always be ascribed to a tension Page |
pneumothorax until proven otherwise. Such pneumothoraces are known to be a complication of
internal jugular venous catheterisation. Immediate decompression is indicated and should be 85
performed prior to any further interventions.

33. A 67-year-old man undergoes carotid endarterectomy having previously suffered a stroke.
During the procedure he sustains an iatrogenic injury to his left vagus nerve. Which of the following
statements about the vagus nerve is the most accurate? Select one answer only.
It enters the abdomen at the level of T10 YOUR ANSWER (Correct)
It exits the skull through the foramen ovale
It lies anterior to the hilum of the lung
It originates from the pons
It travels behind of the oesophagus.

The vagus nerve arises from the medulla and exits the skull through the jugular foramen. It travels
in the carotid sheath between the internal jugular vein and the common carotid artery in the neck. In
the chest it lies posterior to the hilum of the lung, whilst the phrenic nerve travels anterior to the
hilum. The left vagus travels on the anterior aspect of the oesophagus, whilst the right one travels
on the posterior side. Both left and right nerves enter the abdominal cavity at the level of T10
(oesophageal opening).

34. A patient with a malignant mesothelioma is to undergo pleuropneumonectomy, which involves


removal of the entire pleura and lung on the affected side. Which of the following layers provides a
natural cleavage plane for surgical separation of the costal pleura from the thoracic wall? Single
best answer question choose ONE true option only.
Deep fascia YOUR ANSWER
Endothoracic fascia CORRECT ANSWER
Parietal pleura
Visceral pleura
Transversus thoracis muscle fascia.

The endothoracic fascia is the connective tissue between the inner aspect of the chest wall and the
costal parietal pleura. By clearing the endothoracic fascia, it is easy to separate the costal pleura
from the thoracic wall. Deep fascia is a fascial layer that invests a muscle or muscle group - it is not
present around the lungs.

-------------------------------------------------------------------------------------------------------------------------------------------------------------------------
Dr Mohammed Shamsul Islam Khan, Medical Officer, Clinical Neuro-Surgery, National Institute of Neuro-Sciences and Hospital Sher-E-Bangla
Nagar, Dhaka-1207, Bangladesh. Mobile: +880 1713 455 662, E-mail: drsikhan@gmail.com
MyPasTest: MRCS A Online - Jan Exam 2015
02. Anatomy; Thorax(120Qs)
-----------------------------------------------------------------------------------------------------------------
The parietal pleura comprise the cupola or cervical pleura, costal pleura, diaphragmatic pleura and
mediastinal pleura. It lines the inner surfaces of the walls of the pleural cavity. The visceral pleura
are the serous membrane that covers the lungs. Finally, the transversus thoracis muscle fascia is
only associated with the transversus thoracis - it would not provide a natural cleavage plane for
separating the costal pleura from the thoracic wall.

35. A patient presents with a right bundle branch block due to blockage in the atrioventricular nodal Page |
artery. Part of the right bundle branch of the arterioventricular bundle is carried by which structure? 86
Single best answer question choose ONE true option only.
Pectinate muscles YOUR ANSWER
Anterior papillary muscle of the left ventricle
Moderator band (septomarginal trabecula) CORRECT ANSWER
Crista terminalis
Chordae tendineae.
A muscular band, well marked in sheep and some other animals, frequently extends from the base
of the anterior papillary muscle to the ventricular septum. From its attachments it was thought to
prevent overdistension of the ventricle and was named the moderator band. However, more recent
research has indicated that it is more properly considered part of the electrical conduction system
of the heart and in that capacity it is now called the septomarginal trabecula.

36. Following complex surgery for a bypass graft from the right subclavian artery to the brachial
artery, a patient is noted to have a hoarse voice post-operatively. Which of the following structures
is most likely to have been damaged? Select one answer only.
Brachial plexus YOUR ANSWER
Left recurrent laryngeal nerve
Phrenic nerve
Right recurrent laryngeal nerve CORRECT ANSWER
Vagus nerve.

The subclavian artery is closely related to the brachial plexus, the phrenic and vagus nerves (and
recurrent laryngeal nerve on the right side only). The stellate ganglion is deeper within the neck and
the thoracic duct lies on the left side. The approach to the subclavian artery may be supraclavicular
and involve removal or division of the clavicle, dissection of the scalenus anterior, pectoralis major
and pectoralis minor muscles.

37. You are concerned about myocardial ischaemia in a post-operative patient who is complaining
of chest pain. The ECG shows tachycardia and isolated ST-segment depression in leads V1 and V2.
Which coronary artery is most likely to be responsible? Single best answer question choose ONE
true option only.
Circumflex coronary artery YOUR ANSWER
Left anterior descending coronary artery

-------------------------------------------------------------------------------------------------------------------------------------------------------------------------
Dr Mohammed Shamsul Islam Khan, Medical Officer, Clinical Neuro-Surgery, National Institute of Neuro-Sciences and Hospital Sher-E-Bangla
Nagar, Dhaka-1207, Bangladesh. Mobile: +880 1713 455 662, E-mail: drsikhan@gmail.com
MyPasTest: MRCS A Online - Jan Exam 2015
02. Anatomy; Thorax(120Qs)
-----------------------------------------------------------------------------------------------------------------
Posterior descending coronary artery CORRECT ANSWER
Left main stem coronary artery
Right coronary artery.

The coronary artery anatomy is variable, as are the relative territories of myocardium supplied by Page |
each of the arteries. ST-segment depression across all leads implies ischaemia (angina), whereas
isolated ST-segment depression in leads V1 and V2 may indicate a posterior myocardial infarction. 87
The most likely involved artery in this case is therefore the posterior descending artery. Either the
right coronary or circumflex arteries can supply the posterior descending artery, but it is not
possible to identify which from the ECG alone. Left main stem and left anterior descending artery
lesions would cause changes over a much larger area.

38. A 52-year-old morbidly obese diabetic is referred with episodes of chest tightness on minimal
exertion. His exercise ECG shows ST depression, and an angiogram demonstrates left main stem
stenosis. Which of the following best describes the classical anatomy of the left coronary artery?
Select one answer only.
Divides into the circumflex and AV nodal branch YOUR ANSWER
Divides into the circumflex and left anterior descending artery CORRECT ANSWER
Divides into the circumflex and sinoatrial nodal branch
Divides into the posterior descending artery and anterior descending artery
Divides into the posterior descending artery and marginal artery.

The left coronary artery arises from the left posterior aortic sinus behind the pulmonary trunk. After
a short course it divides into two main arteries, the circumflex and the left anterior descending,
otherwise known as the anterior interventricular artery.

Around 60% of hearts have the right coronary artery supplying the SA node and in 40% of hearts
the SA nodal artery arises from the left coronary artery. The right coronary artery supplies the
atrioventricular (AV) node. The left coronary artery supplies the vast majority of the left ventricle
and left atrium. Part of the right ventricle is supplied by the left coronary artery.

39. A 67-year-old male who is a smoker presents with central chest pain radiating to his neck. His
ECG shows ST elevation in chest leads V5 and V6. A coronary angiography shows occlusion of the
circumflex artery and, following successful angiography, this can be seen to give rise to the
posterior interventricular branch. In approximately what proportion of people is this pattern seen?
Select one answer only.
1% YOUR ANSWER
15% CORRECT ANSWER
30%
60%
90%.

-------------------------------------------------------------------------------------------------------------------------------------------------------------------------
Dr Mohammed Shamsul Islam Khan, Medical Officer, Clinical Neuro-Surgery, National Institute of Neuro-Sciences and Hospital Sher-E-Bangla
Nagar, Dhaka-1207, Bangladesh. Mobile: +880 1713 455 662, E-mail: drsikhan@gmail.com
MyPasTest: MRCS A Online - Jan Exam 2015
02. Anatomy; Thorax(120Qs)
-----------------------------------------------------------------------------------------------------------------
The interventricular septum receives its blood supply from the descending branches from the left
coronary (LCA) and right coronary arteries (RCA). RCA travels in the coronary groove to reach the
posterior surface of the heart where it anastomoses with the circumflex branch of LCA.

Posterior interventricular artery, that is a branch of RCA, anastomoses with the anterior
interventricular artery in the interventricular groove. In 15% of hearts the LCA is said to be
dominant in that the posterior interventricular branch comes off the circumflex branch. Page |
88
40. A 27-year-old male is brought into A and E following a stabbing to his left chest. Absent air
entry is noted on examination of his left chest and it is dull to percussion. The diagnosis of a
haemothorax is made and a chest drain placed. This drains 1600ml of blood immediately and the
patient becomes more hypotensive. What should be the next step in managing the patient? Select
one answer only.
Blood transfusion and correct any coagulopathy YOUR ANSWER
Clamp the drain
Emergency thoracotomy CORRECT ANSWER
Inotropes to improve the blood pressure
Midline sternotomy.

Clamping of chest drains is largely to be avoided and achieves nothing clinically. Massive bleed is
an immediate indication for urgent thoracotomy for surgical haemostasis
(http://www.trauma.org/archive/thoracic/EDTindications.html ). Any haemodynamic instability should
lead to consideration of surgical management.

Persistent haemorrhage of at least 200 ml/h for four or more hours is an indication for thoracotomy.
Video-assisted thoracoscopy may be useful to establish a source of bleeding. Surgical intervention
is best undertaken in the operating theatre a controlled environment, aseptic, adequate light etc.
Outcomes from emergency room thoracotomy are poor. Retained blood may cause an empyema.

41. A 61-year-old heavy smoker presents with recurrent chest infections treated with antibiotics
from the GP. A mass is seen on his chest X-ray and a bronchoscopy confirms this is malignant in
nature. Which of the following is a contraindication to resection? Select one answer only.
COPD YOUR ANSWER
FEV/FVC of 70%
Ipsilateralhilar lymphadenopathy
Malignant pleural effusion CORRECT ANSWER
Ongoing smoking status.

A lung tumour must be sufficiently localised to be suitable for resection. Involvement of


ipsilateralhilar lymph nodes is not usually a contraindication to resection but the presence of
malignant cells in a pleural effusion is. The 5-year survival rate following complete resection of non-
small-cell lung cancer is in the region of 3040%.

-------------------------------------------------------------------------------------------------------------------------------------------------------------------------
Dr Mohammed Shamsul Islam Khan, Medical Officer, Clinical Neuro-Surgery, National Institute of Neuro-Sciences and Hospital Sher-E-Bangla
Nagar, Dhaka-1207, Bangladesh. Mobile: +880 1713 455 662, E-mail: drsikhan@gmail.com
MyPasTest: MRCS A Online - Jan Exam 2015
02. Anatomy; Thorax(120Qs)
-----------------------------------------------------------------------------------------------------------------
Small-cell lung cancers (also known as oat-cell carcinoma) are highly malignant tumours that are
usually disseminated at presentation. For the majority of patients chemotherapy is the treatment of
choice, small-cell lung cancers are rarely suitable for surgical management.

42. A 35-year-old woman is noted to have a left phrenic nerve injury after excision of a left cervical
rib. Which of these descriptions best describes the relation of the phrenic nerve in this area? Select
one answer only. Page |
Anterior to scalenus anterior YOUR ANSWER (Correct) 89
Between scalenus anterior and scalenus medius
Medial to scalenus anterior
Posterior to scalenus medius
Posterior to scalenus posterior.

The phrenic nerve arises from the spinal cord segments C35 and lies in front of the scalenus
anterior muscle, passing between the subclavian vein anteriorly and the subclavian artery
posteriorly. It crosses over the lateral surfaces of the mediastinal structures passing in front of the
lung root to reach the diaphragm.

The phrenic nerve supplies motor fibres to the diaphragm and carries sensory fibres from the
diaphragmatic peritoneum, mediastinal pleura and the parietal pericardium. The bronchi and lungs
themselves are supplied by branches of the autonomic nerves, principally via the pulmonary
plexuses.

43. A 56-year-old diabetic male is undergoing a CABG for left main stem disease following an MI.
Which of the following terms best describes the anaesthetic approach to cardiac surgery of this
nature? Select one answer only.
Controlled hypertension, controlled hyperthermia YOUR ANSWER
Controlled hypertension, controlled hypothermia
Controlled hypotension, controlled hypothermia CORRECT ANSWER
Controlled hypotension, controlled normothermia
Controlled normotension, controlled hypothermia.

Cardiac surgery is performed with controlled hypotension, together with hypothermia. The
hypothermia is used to decrease cellular metabolism and reduce energy requirements of the
tissues. Cardiac tamponade is a well known complication of cardiac surgery, it usually presents in
the early post-operative period with deteriorating cardiac function and cardiac arrest.

CABG can sometimes be performed without cardiopulmonary bypass (off pump). Arrhythmias
(usually atrial fibrillation) occur in 2040% of patients following surgery. Routine CABG is
associated with a mortality rate of around 2%.

-------------------------------------------------------------------------------------------------------------------------------------------------------------------------
Dr Mohammed Shamsul Islam Khan, Medical Officer, Clinical Neuro-Surgery, National Institute of Neuro-Sciences and Hospital Sher-E-Bangla
Nagar, Dhaka-1207, Bangladesh. Mobile: +880 1713 455 662, E-mail: drsikhan@gmail.com
MyPasTest: MRCS A Online - Jan Exam 2015
02. Anatomy; Thorax(120Qs)
-----------------------------------------------------------------------------------------------------------------
44. A 23-year-old motorcyclist presents with a blunt chest injury and is noted to have
hyperresonance and absent breath sounds over the right hemithorax with evidence of tracheal
deviation to the left. What should be the first clinical intervention?
Endotracheal intubation YOUR ANSWER
Intercostal drainage
Page |
Needle decompression CORRECT ANSWER
90
Positive end-expiratory pressure ventilation
Thoracotomy.

This patient has a right-sided tension pneumothorax. If such a diagnosis is suspected, immediate
needle decompression should be performed by inserting a wide bore cannula into the second
intercostal space in the mid-clavicular line of the affected hemithorax. This buys time for the more
definitive intervention of intercostal drain insertion. Positive end-expiratory pressure ventilation is
known to be a risk factor for the development of a tension pneumothorax. Thoracotomy is rarely
indicated in cases of tension pneumothorax.

45. A 19-year-old male gang member is brought into A&E after being stabbed multiple times in the
right chest. He is drowsy and has a BP on arrival of 70/40 with cool peripheries. The right side of his
chest is dull to percussion with absent breath sounds. A chest drain is placed in his right chest and
drains 1700ml within 15 minutes. What is the diagnosis? Select one answer only.
Haemothorax YOUR ANSWER
Haemopneumothorax
Massive haemothorax CORRECT ANSWER
Pneumothorax
Tension pneumothorax.

A haemothorax may result from a blunt (deceleration injury) or penetrating injury (disruption of the
systemic or hilar vessels) to the thoracic cavity. Massive haemothorax results from the rapid
accumulation of more than 1500 ml of blood or one-third or more of the patients blood volume in
the chest cavity. Distension of neck veins is rare; they are usually flat secondary to severe
hypovolemia.

Rarely will the mechanical effects of massive intrathoracic blood shift the mediastinum enough to
cause distended neck veins or a tracheal shift. The neck veins, however, may be distended if there
is an associated tension pneumothorax. The important signs and symptoms of a massive
haemothorax include hypoxia, chest pain, decreased chest expansion, absence of breath sounds
on the affected side and percussion dullness over the affected hemithorax. Chest tube placement to
decompress the chest cavity, along with simultaneous restoration of blood volume, is the first step
in the management of massive traumatic haemothorax.

Blood from the chest tube should be collected in a device suitable for auto-transfusion. If 1500 ml is
immediately evacuated, it is highly likely that the patient will require an early thoracotomy. In

-------------------------------------------------------------------------------------------------------------------------------------------------------------------------
Dr Mohammed Shamsul Islam Khan, Medical Officer, Clinical Neuro-Surgery, National Institute of Neuro-Sciences and Hospital Sher-E-Bangla
Nagar, Dhaka-1207, Bangladesh. Mobile: +880 1713 455 662, E-mail: drsikhan@gmail.com
MyPasTest: MRCS A Online - Jan Exam 2015
02. Anatomy; Thorax(120Qs)
-----------------------------------------------------------------------------------------------------------------
addition, patients who have an initial volume output of less than 1500 ml but continue to bleed (200
ml/h for 24 h) also require a thoracotomy.

The decision should be made in such patients based on their physiological status rather than the
rate of continuing blood loss. The majority of the patients can be managed conservatively with
appropriate fluid resuscitation and chest decompression. Emergency thoracotomy is required in
only about 10% of patients with massive haemothorax. Page |
91
46. A 32-year-old male is rushed into A & E following an explosion at a factory. He has severe facial
burns and requires a needle cricothyroidotomy following a failed intubation. Following stabilisation,
it is decided to perform a tracheostomy. Which of the following landmarks represents the origin of
the trachea? Select one answer only.
Cricoid cartilage YOUR ANSWER (Correct)
Hyoid bone
Jugular notch
Laryngeal prominence
Sternal angle.

The trachea commences at the lower border of the cricoid cartilage, is palpable in the jugular notch,
bifurcates in the transthoracic plane and is innervated by the recurrent laryngeal nerves. The left
main bronchus bifurcates inside the left lung and is not as vertical as the right main bronchus.

47. A 41-year-old male is brought into A & E very short of breath following a road traffic accident.
He is unable to complete sentences and has a respiratory rate of 34. On examination there is
bruising over his right chest and chest expansion on the right is reduced with tracheal deviation
away from this side. Percussion is resonant and breath sounds are normal on the left, but on the
right percussion is hyper-resonant and breath sounds are absent. What is the most likely
diagnosis? Select one answer only.
Flail chest YOUR ANSWER
Haemothorax
Pneumothorax
Tension pneumothorax CORRECT ANSWER
Chylothorax.

Tension pneumothorax is one of the immediate life-threatening injuries and must be diagnosed and
managed very early. It is recognised by respiratory distress, tracheal deviation away from the
affected side, unilateral absent breath sounds, distended neck veins and not always accompanied
by rib fractures.

48. A 73-year-old man is found to have a thoracic aortic aneurysm affecting the ascending aorta and
aortic arch. The aneurysm is expanding in size, leading to aortic regurgitation. Which of the
following best describes the anatomy of the normal thoracic aortic arch? Select one answer only.
-------------------------------------------------------------------------------------------------------------------------------------------------------------------------
Dr Mohammed Shamsul Islam Khan, Medical Officer, Clinical Neuro-Surgery, National Institute of Neuro-Sciences and Hospital Sher-E-Bangla
Nagar, Dhaka-1207, Bangladesh. Mobile: +880 1713 455 662, E-mail: drsikhan@gmail.com
MyPasTest: MRCS A Online - Jan Exam 2015
02. Anatomy; Thorax(120Qs)
-----------------------------------------------------------------------------------------------------------------
It arches below the manubriosternal joint YOUR ANSWER
It arches directly over the right pulmonary artery
It gives rise to the coronary arteries
It is covered by pleura CORRECT ANSWER
Page |
It lies anterior to the brachiocephalic veins.
92
The apex of the arch, which gives attachment to the pretracheal fascia, lies posteroinferior to the
left brachiocephalic vein. The lower border of the arch lies in the transthoracic plane and on the left
is directly related to the left pulmonary artery (the superior vena cava lies over the right pulmonary
artery). The arch is symmetrically covered by the pleura from both sides, which meet in the midline
behind the manubriosternal joint. The coronary arteries are the first branches of the ascending
aorta and not branches of the aortic arch. This gives rise to the brachiocephalic artery, the left
common carotid and the left subclavian artery.

49. During angiography a 56-year-old diabetic patient is noted to have stenosis of the posterior
interventricular artery. Which of the following most often gives rise to this vessel? Select one
answer only.
Anterior interventricular artery YOUR ANSWER
Circumflex artery
Left coronary artery
Marginal branch of the right coronary artery
Right coronary artery CORRECT ANSWER.

The right coronary artery runs in the atrioventricular groove between the right atrium and right
ventricle. It has no branches which directly supply the ventricular septum. The posterior
descending artery most commonly arises from the right coronary artery, and can be found
overlying the interventricular septum posteriorly.

It may arise as a branch of the circumflex artery, but not as a branch of the left anterior descending
artery. The main circumflex artery does lie in the atrioventricular groove, and its obtuse marginal
branches supply the lateral wall of the left ventricle. The acute marginal branch arises from the right
coronary artery.

50. A 72-year-old smoker undergoes a resection of the right middle lung lobe for squamous cell
lung cancer. The lesion is adherent to the mediastinum but is removed successfully. A post-
operative chest x-ray reveals a raised right hemi-diaphragm. There is no evidence of a
pneumothorax. Which of the following structures is most likely to have been damaged? Select one
answer only.
The liver capsule and parenchyma YOUR ANSWER
The phrenic nerve CORRECT ANSWER

-------------------------------------------------------------------------------------------------------------------------------------------------------------------------
Dr Mohammed Shamsul Islam Khan, Medical Officer, Clinical Neuro-Surgery, National Institute of Neuro-Sciences and Hospital Sher-E-Bangla
Nagar, Dhaka-1207, Bangladesh. Mobile: +880 1713 455 662, E-mail: drsikhan@gmail.com
MyPasTest: MRCS A Online - Jan Exam 2015
02. Anatomy; Thorax(120Qs)
-----------------------------------------------------------------------------------------------------------------
The right hemi-diaphragm
The thoracic duct
The vagus nerve.

The phrenic nerve (C3-5) provides sensory and motor fibres to the diaphragm, as well as fibres to Page |
the pleura and mediastinal pericardium. An injury of the phrenic nerve causes paralysis of the
corresponding hemidiaphragm. This leads to paradoxical movement of the diaphragm, rising on 93
inspiration. The phrenic nerve may be injured in any surgical procedure in the neck (where it lies on
the anterio scalene muscle) or near the mediastinum. Spinal cord injuries above or at the level of
C3-C5 may lead to respiratory compromise. Remember 'C3, 4 and 5 keep the diaphragm alive'.

A raised hemidiaphragm may also be caused by a subphrenic abscess or more rarely by a


subcapsular haematoma of the liver. In the context of recent thoracic surgery, the most likely cause
is iatrogenic phrenic nerve damage during the tumour dissection.

51. A 63-year-old long term smoker presents with haemoptysis and weight loss. A chest X-ray
shows a tumour and bronchoscopy reveals it to be at the carina. At what level is the carina? Select
one answer only.
Level with xiphoid process YOUR ANSWER
Jugular notch
Sternal angle of Louis CORRECT ANSWER
Suprasternal notch
Transpyloric plane.

The brachiocephalic artery bifurcates behind the right sternoclavicular joint. The ascending aorta
becomes the aortic arch at the manubriosternal joint (sternal angle of Louis). A plane passing
posteriorly will reach the lower border of the T4 vertebrae dividing the mediastinum into the
superior mediastinum and inferior mediastinum. The trachea bifurcates at the carina into the two
main stem bronchi at this level. At this level, the azygous, the left recurrent laryngeal nerve looping
around the ligamentumarteriosum, and the bifurcation of the pulmonary trunk, can also be seen.

52. A 71-year-old heavy smoker is listed for a left posterolateral thoracotomy for resection of a
tumour. How should he be positioned on the table? Select one answer only.
Left lateral position YOUR ANSWER
Prone
Reverse trendelenburg
Right lateral position CORRECT ANSWER
Supine.

Posterolateral thoracotomy is performed most commonly with the patient in a full lateral position.
The skin incision extends from a point two fingerbreadths below the inferior angle of the scapula,

-------------------------------------------------------------------------------------------------------------------------------------------------------------------------
Dr Mohammed Shamsul Islam Khan, Medical Officer, Clinical Neuro-Surgery, National Institute of Neuro-Sciences and Hospital Sher-E-Bangla
Nagar, Dhaka-1207, Bangladesh. Mobile: +880 1713 455 662, E-mail: drsikhan@gmail.com
MyPasTest: MRCS A Online - Jan Exam 2015
02. Anatomy; Thorax(120Qs)
-----------------------------------------------------------------------------------------------------------------
posteriorly to a point halfway between the spine of the scapula and the vertebral column, and
anteriorly following the line of the rib for a variable distance.

The latissimus dorsi is routinely divided but the serratus anterior is usually spared. The chest is
entered through the intercostal spaces, which are counted as starting below the first rib. Thus the
fifth space lies below the fifth rib. The intercostal muscles are divided from the superior border of Page |
the rib below the intercostal space, to be entered to avoid injury to the neurovascular bundle lying 94
just inferior to each rib.

53. The sinus venosus is the large quadrangular cavity located between the two vena cava vessels
in the embryonic human heart. In the adult heart the sinus venosus gives rise to the: Single best
answer question choose ONE true option only.
Aortic vestibule of the left ventricle YOUR ANSWER
Coronary sinus CORRECT ANSWER
Fossa ovalis
Trabeculated part of the right ventricle
Trabeculated portion of the right atrium.

The derivatives of the embryonic sinus venosus include: the smooth part of the right atrium (sinus
venarum), the valve of the superior vena cava and the sinoatrial node from the right horn; the
coronary sinus and the valve of coronary sinus from the left horn; the border of smooth part of
right atrium (crista terminalis) from the right half of the valve of the sinus venosus and part of the
atrial septum from the left half of the valve of the sinus venosus.

54. You are asked by your consultant to perform a median sternotomy under supervision. During
the manoeuver you are asked to describe which vascular structures are positioned on the same
anatomical plane as the manubriosternal joint. What is the correct answer?
Aortic arch and junction of the azygos vein and hemiazygos vein YOUR ANSWER
Aortic arch and junction of the azygos vein and superior vena cava CORRECT ANSWER
Descending thoracic aorta and junction of the azygos and superior vena cava
Hemiazygos vein
Origin of the brachiocephalic artery.

The manubriosternal joint marks the thoracic plane, which is at the level of the T4/5 intervertebral
disc. Other structures found on this plane include the aortic arch, the junction of the azygos vein
with the SVC, the 2nd costal cartilages, the tracheal bifurcation, the ligamentum arteriosum and the
decussation of the thoracic duct. The manubriosternal joint is also known as the Angle of Louis.

55. A 78-year-old male being worked up for an open abdominal aortic aneurysm repair undergoes a
routine chest X-ray which shows a widened mediastinum. Further imaging confirms a thoracic
aneurysm affecting the aortic arch. Which of the following branches normally arises from the aortic
arch? Select one answer only.
-------------------------------------------------------------------------------------------------------------------------------------------------------------------------
Dr Mohammed Shamsul Islam Khan, Medical Officer, Clinical Neuro-Surgery, National Institute of Neuro-Sciences and Hospital Sher-E-Bangla
Nagar, Dhaka-1207, Bangladesh. Mobile: +880 1713 455 662, E-mail: drsikhan@gmail.com
MyPasTest: MRCS A Online - Jan Exam 2015
02. Anatomy; Thorax(120Qs)
-----------------------------------------------------------------------------------------------------------------
Left internal mammary artery YOUR ANSWER
Left subclavian artery CORRECT ANSWER
Right common carotid artery
Right subclavian artery
Page |
Right vertebral artery.
95
The arch of the aorta commences from the manubriosternal joint and passes backwards over the
left bronchus to reach the body of T4 vertebra just to the left of the midline.

The branches arising directly from the aortic arch are the brachiocephalic trunk (which will divide
into the right subclavian artery and right common carotid artery), the left common carotid artery
and left subclavian artery.

The arch is crossed on its left side by the phrenic and vagus nerves as they pass downwards in
front of and behind the lung root respectively. The left vertebral artery and the right internal
mammary artery come off the left and right subclavian arteries respectively.

56. A 21-year-old man is brought into A&E unresponsive after being stabbed in the chest. On
examination the knife has been left in place and is entering through the 4th intercostal space on the
left of the sternum. He has a heart rate of 140 and his BP is 70/30. He is noted to have a central
trachea, distended neck veins. He has normal breath sounds bilaterally, but muffled heart sounds.
What is the most likely diagnosis? Select one answer only.
Aortic dissection YOUR ANSWER
Cardiac tamponade CORRECT ANSWER
Haemothorax
Pneumothorax
Tension pneumothorax.

ECG is useful to detect dysrhythmias and pulseless electrical activity (PEA). There is no indication
for routine exploratory thoracotomy within 1 hour. An urgent exploratory thoracotomy should be
performed if 1500 ml of blood is drained immediately from the chest drain or 200 ml/h for 24 hours.
A normal appearance of the cardiac shadow on CXR is insufficient to exclude a tamponade.

Echocardiography and computed tomography (CT) are more sensitive methods. Neck veins should
be assessed for distension. Becks triad for cardiac tamponade consists of muffled heart sounds,
hypotension and distended neck veins. Tension pneumothorax is a clinical diagnosis requiring
immediate management.

Aortic dissection classically causes a tearing chest pain radiating through to the back. On
examination, the patient can have a variety of symptoms depending on which part of the aorta (and
therefore who branches) are affected. If the common carotid artery is affected they may have a
neurological signs, there may be signs in the upper limb if the subclavian arteries are affected.

-------------------------------------------------------------------------------------------------------------------------------------------------------------------------
Dr Mohammed Shamsul Islam Khan, Medical Officer, Clinical Neuro-Surgery, National Institute of Neuro-Sciences and Hospital Sher-E-Bangla
Nagar, Dhaka-1207, Bangladesh. Mobile: +880 1713 455 662, E-mail: drsikhan@gmail.com
MyPasTest: MRCS A Online - Jan Exam 2015
02. Anatomy; Thorax(120Qs)
-----------------------------------------------------------------------------------------------------------------
If the dissection involves the abdominal aorta, the patient may have abdominal signs or oliguria/
anuria if the renal arteries are affected. Classically, there may be a difference in blood pressure
between the arms due to subclavian artery involvement on one side.

A haemothorax would cause chest signs on examination- reduced or absent air/entry and dullness
to percussion, this helps to differentiate it from a pneumothorax in which air entry may also be
reduced or absent, but percussion is hyper-resonant. In tension pneumothorax a valve like Page |
mechanism allows the accumulation of air in the pleural space which causes medistinal 96
displacement- thus affecting venous return to the heart and therefore cardiac output.

It will lead to severe shortness of breath, tracheal deviation, dilated neck veins and ultimately
cardiac arrest. The diagnosis is clinical, and management should not be delayed for a chest X ray.

57. An 82-year old man develops hilar lymphadenopathy secondary to a haematological


malignancy. This affects predominantly the superior aspect of the right main bronchus. Which of
the following structures is most likely to be compressed? Select one answer only.
The azygos vein YOUR ANSWER (Correct)
The hemiazygos nerve
The right phrenic nerve
The right recurrent laryngeal nerve
The right vagus nerve.

The vagus nerve lies just posterior to the right main bronchus and the azygos vein is at first
posterior and then arches over the superior aspect of the right main bronchus to join the SVC. The
phrenic nerve is anterior to the bronchus. The right recurrent laryngeal nerve hooks around the
right subclavian artery superior to the right main bronchus.

58. A 78-year-male with severe aortic stenosis undergoes coronary artery bypass grafting for
coronary artery disease whilst having an aortic valve replacement. Which is the most commonly
used arterial conduit? Select one answer only.
Inferior epigastric artery YOUR ANSWER
Left internal mammary artery CORRECT ANSWER
Long saphenous vein
Right internal mammary artery
Radial artery.

The most commonly used arterial conduit is the left internal mammary artery. Currently the right
internal mammary artery and radial artery are also in use by some surgeons. The gastroepiploic
and inferior epigastric arteries have been used, but are uncommon bypass conduits.

-------------------------------------------------------------------------------------------------------------------------------------------------------------------------
Dr Mohammed Shamsul Islam Khan, Medical Officer, Clinical Neuro-Surgery, National Institute of Neuro-Sciences and Hospital Sher-E-Bangla
Nagar, Dhaka-1207, Bangladesh. Mobile: +880 1713 455 662, E-mail: drsikhan@gmail.com
MyPasTest: MRCS A Online - Jan Exam 2015
02. Anatomy; Thorax(120Qs)
-----------------------------------------------------------------------------------------------------------------
59. A victim of anterior chest stabbing received a stab in a structure that is in close proximity to
where the first rib articulates with the sternum. The structure most likely to be injured is the: Single
best answer question choose ONE true option only.
Nipple YOUR ANSWER
Root of the lung
Page |
Sternal angle
97
Sternoclavicular joint CORRECT ANSWER
Xiphoid process.

The first rib articulates with the sternum directly below the sternoclavicular joint. The nipple (in
men) may be considered to be in the fourth intercostal space, between the fourth and fifth ribs, but
this is an inconsistent surface landmark due to anatomical variation (eg the presence of breasts in
women). The sternal angle is connected to the costal cartilage of rib 2.

Finally, the xiphoid process is located just below the point where the costal cartilage of rib 7
articulates with the sternum. The root of the lung consists of the main bronchus, pulmonary and
bronchial vessels, lymphatic vessels and nerves entering and leaving the lung. The roots of the
lungs are anterior to vertebrae T5 (right) and T6 (left).

60. A 48-year-old female smoker presents with acute severe chest pain and ST elevation on the
ECG. Troponin levels are raised and suggestive of myocardial infarction. She undergoes Primary
Coronary Angioplasty and is noted to have an occlusion within the vessel running through the right
coronary sulcus. Which vessel has been occluded?
Conus artery YOUR ANSWER
Left circumflex artery
Left coronary artery
Posterior descending artery
Right coronary artery CORRECT ANSWER.

The right coronary artery originates from the ascending aorta just distal to the right cusp of the
aortic valve. It travels within the right coronary sulcus and gives off the posterior descending artery
and sinoatrial node artery in most cases. The conus artery is present in 45% of the population and
can provide collateral circulation in cases of left anterior descending artery occlusion.

61. In a post-operative immunosuppressed transplant patient, some small vesicles have formed on
the chest wall, in a linear patch, approximately level with his xiphoid process. Which dermatome is
most likely to be affected? Single best answer question choose ONE true option only.
C4 YOUR ANSWER
T4
T7 CORRECT ANSWER
T10
-------------------------------------------------------------------------------------------------------------------------------------------------------------------------
Dr Mohammed Shamsul Islam Khan, Medical Officer, Clinical Neuro-Surgery, National Institute of Neuro-Sciences and Hospital Sher-E-Bangla
Nagar, Dhaka-1207, Bangladesh. Mobile: +880 1713 455 662, E-mail: drsikhan@gmail.com
MyPasTest: MRCS A Online - Jan Exam 2015
02. Anatomy; Thorax(120Qs)
-----------------------------------------------------------------------------------------------------------------
L1.
C4 corresponds to the lower neck just above the clavicle. T4 corresponds to the level of the
nipples. T7 is the level of the xiphoid process. T10 corresponds to the level of the umbilicus, and L1
corresponds to the lower abdomen / inguinal region.

62. A 22-year-old male is rushed into A & E after being stabbed in the chest following an assault.
Page |
The knife is still in place and he is taken to theatre for urgent surgical exploration. The knife is 98
found to have penetrated the anterior wall of the right atrium. Which of the following structures are
present within the anterior wall of the right atrium? Select one answer only.
Conusarteriosus YOUR ANSWER
Coronary sinus
Musculipectinati CORRECT ANSWER
Oval fossa
Sinus venarum.

The right atrium forms the right border of the heart between the superior and inferior vena cava.
The internal walls consist of a smooth part posteriorly, the sinus venarum, which receives the
venae cavae and coronary sinus and a rough anterior portion, which has the musculipectinati. The
conusarteriosus is found in the right ventricle.

63. A 32-year-old male who is involved in a road traffic accident whilst driving his motorbike has a
pneumothorax clinically which is confirmed on imaging. It is decided he requires a chest drain.
Which of the following best describes a safe location for insertion? Select one answer only.
Posterior to the mid-axillary line, 4th intercostal space below 4th rib YOUR ANSWER
Posterior to the mid-axillary line, 4th intercostal space above 5th rib
Anterior to the mid-axillary line, 4th intercostal space below 4th rib
Anterior to the mid-axillary line, 4th intercostal space above 5th rib CORRECT ANSWER
2nd intercostal space, midclavicular line.

For the anatomy of chest drain insertion, it is important to remember the 'safe triangle.' This is
bordered by the anterior border of the latissimus dorsi (mid-axillary line), the lateral border of the
pectoralis major muscle, a line superior to the horizontal level of the nipple (5th intercostal space),
and an apex below the axilla.

For insertion, the area should be draped, and patient consented (as per BTS guidelines). Local
anaesthestics can then be infiltrated over the area and a 2cm incision made ABOVE the rib (this is
to minimise the risk of damage to the neurovascular bundle), and blunt dissection performed down
to the pleura. Forceps or scissors can be used to pierce the pleura and a finger should be swept
around inside prior to drain insertion to remove any adherent lung tissue.

-------------------------------------------------------------------------------------------------------------------------------------------------------------------------
Dr Mohammed Shamsul Islam Khan, Medical Officer, Clinical Neuro-Surgery, National Institute of Neuro-Sciences and Hospital Sher-E-Bangla
Nagar, Dhaka-1207, Bangladesh. Mobile: +880 1713 455 662, E-mail: drsikhan@gmail.com
MyPasTest: MRCS A Online - Jan Exam 2015
02. Anatomy; Thorax(120Qs)
-----------------------------------------------------------------------------------------------------------------
The drain can then be inserted (without the trocar). When placing a chest drain it should be pointed
upward if a pneumothorax is being drained, and downward if fluid e.g. a pleural effusion is being
drained.

Alternatively some kits nowadays utilise the Seldinger technique for insertion.

Following insertion chest drains should be connected with an appropriate underwater seal. It is Page |
important to note, they should never be clamped. 99
The 2nd intercostal space, midclavicular line is utilised when performing a needle decompression of
a tension pneumothorax to relieve the tensioning. Following this a formal chest drain can be
inserted as described above.

64. A 48-year-old woman presents with muscle weakness and problems chewing food. Her
symptoms appear to be worsened on repetition and on examination she is noted to have bilateral
ptosis, and a diagnosis of myasthenia gravis is made. A CT thorax shows an anterior mediastinal
tumour. What is likely to be the nature of this? Select one answer only.
Germ cell tumour YOUR ANSWER
Lymphoma
Mesothelioma
Small cell lung cancer
Thymoma CORRECT ANSWER.

Thymoma is the commonest anterior mediastinal tumour, although it is still rare in everyday clinical
practice. There is a definite association with myasthenia gravis. The differential diagnosis for an
anterior mediastinal mass is lymphoma and germ cell tumour. It is germ cell tumours that may
cause an elevated serum b-human chorionic gonadotrophin (b-HCG).

Any mass in the anterior mediastinum may compress the superior vena cava leading to the
superior vena cava obstruction syndrome. Needle biopsy is still not useful in differentiating benign
from malignant thymic tumours, but is fairly reliable in differentiating these tumours from
lymphoma and germ cell tumours.

65. A 27-year-old female is brought into A & E as a trauma call after a horse riding accident in which
the horse she was riding fell backward landing on the left side of her chest. On arrival she is short
of breath and struggling to complete sentences. On examination the chest wall containing ribs 3-5
appears to move in on inspiration, air entry is present bilaterally and her trachea is central. What is
the most likely diagnosis? Select one answer only.
Flail chest YOUR ANSWER (Correct)
Haemothorax
Pneumothorax
Surgical emphysema
-------------------------------------------------------------------------------------------------------------------------------------------------------------------------
Dr Mohammed Shamsul Islam Khan, Medical Officer, Clinical Neuro-Surgery, National Institute of Neuro-Sciences and Hospital Sher-E-Bangla
Nagar, Dhaka-1207, Bangladesh. Mobile: +880 1713 455 662, E-mail: drsikhan@gmail.com
MyPasTest: MRCS A Online - Jan Exam 2015
02. Anatomy; Thorax(120Qs)
-----------------------------------------------------------------------------------------------------------------
Tension pneumothorax.

Ribs are narrow, flat, curved bones that form the thoracic cage. The rib has an angle at its posterior
end. The costal groove is found on the inferior border of the ribs. The intercostal vessels and nerve
run in this groove. Ribs typically articulate with their corresponding vertebra plus the one above it.
The area just anterior to the angle is the weakest, and therefore the most likely to fracture in Page |
trauma. 100
66. A 34-year-old, tall man presents with sudden onset shortness of breath. He is noted to have
absent breath sounds in the left hemi-thorax. A chest x-ray confirms a spontaneous simple left
pneumothorax and chest drain is inserted. When inserting an intercostal drain in the 5th intercostal
space. Which of the following structures is normally breached by the chest tube? Select one
answer only.
Parietal pleura YOUR ANSWER (Correct)
Pectoralis major muscle
Pectoralis minor muscle
Pericardium
Visceral pleura.

An intercostal drain should be inserted in the fifth intercostal space, anterior to the mid-axillary line
and so should not penetrate pectoralis major. The visceral pleura overlies the lung and should not
be entered. After piercing the skin, fat and intercostals muscles, the chest drain must pass through
the parietal pleura lining the inner thorax. This allows the drain to reach the interpleural space
(found between the parietal and visceral pleurae), which allows the pleural cavity to be drained.

67. After a difficult right-sided subclavian central line insertion, which is the least likely structure to
be damaged? Single best answer question choose ONE true option only.
Thoracic duct YOUR ANSWER (Correct)
Brachial plexus
Subclavian vein
Subclavian artery
Lung apex.

The thoracic duct has usually crossed over to the left side by the level of the 5 th or 6th vertebral
body, and is therefore the furthest structure from the advancing needle. The brachial plexus,
subclavian artery and lung can all be reached and damaged by a needle during subclavian
cannulation. The subclavian vein, although the target vessel in this case (and therefore intentionally
punctured) can be damaged (torn) during line insertion.

68. A 68-year-old heavy smoker presents to his GP with a drooping left eyelid and dryness affecting
the left side of his face. His GP notes he has a left sided ptosis, miosis and anhidrosis, and a
-------------------------------------------------------------------------------------------------------------------------------------------------------------------------
Dr Mohammed Shamsul Islam Khan, Medical Officer, Clinical Neuro-Surgery, National Institute of Neuro-Sciences and Hospital Sher-E-Bangla
Nagar, Dhaka-1207, Bangladesh. Mobile: +880 1713 455 662, E-mail: drsikhan@gmail.com
MyPasTest: MRCS A Online - Jan Exam 2015
02. Anatomy; Thorax(120Qs)
-----------------------------------------------------------------------------------------------------------------
diagnosis of Horner syndrome is made. A CT scan shows a Pancoast tumour invading the stellate
ganglion. Which of the following descriptions best describes the relations of the stellate ganglion?
Select one answer only.
Anterior to the neck of the first rib YOUR ANSWER (Correct)
Anterior to the scalene tubercle
Page |
Lateral to the angle of the first rib
101
Posterior to the neck of the first rib
Posterior to the scalene tubercle.

The scalenus anterior inserts on the scalene tubercule. A groove is found on the first rib anterior to
the scalene tubercule for the subclavian vein. The subclavian artery runs behind the scalenus
anterior muscle. The first rib is related to the lower two roots of the brachial plexus, C8 and T1. The
cervicothoracic ganglion, otherwise known as the stellate ganglion, lies in front of the neck of the
first rib.

69. A 21-year old man is shot in the chest. He is diagnosed with a pericardial tamponade. He
undergoes an emergency thoracotomy and creation of a pericardial window by excising a segment
of pericardium to relieve the tamponade. Which of the following can occur as result of an iatrogenic
injury during the pericardial incision? Select one answer only.
Damage to the parasympathetic supply to the abdomen YOUR ANSWER
Elevation of the left hemidiaphragm CORRECT ANSWER
Hiatal hernia
Paralysis of all intercostals muscles on the left hemithorax
Pulsion diverticulum of the oesophagus.

The phrenic nerve (C3 5) supplies motor innervation to the diaphragm. It runs vertically on the
external surface of the cardiac pericardium. An incision over the pericardium in emergency
situations can lead to a phrenic nerve damage and paralysis of the ipsilateral hemidiaphragm.

70. A 25-year-old female presents with pain in the right hand which is associated with a tingling
sensation and weakness. After further investigation she is diagnosed with thoracic outlet
syndrome. The affected neurovascular bundle runs through which muscles?
Digastric YOUR ANSWER
Platysma
Scalene CORRECT ANSWER
Sternocleidomastoid
Sternothyroid.

Thoracic outlet syndrome is caused by compression of the neurovascular bundle that passes
between the anterior and middle scalene muscles. The brachial plexus and the subclavian artery
-------------------------------------------------------------------------------------------------------------------------------------------------------------------------
Dr Mohammed Shamsul Islam Khan, Medical Officer, Clinical Neuro-Surgery, National Institute of Neuro-Sciences and Hospital Sher-E-Bangla
Nagar, Dhaka-1207, Bangladesh. Mobile: +880 1713 455 662, E-mail: drsikhan@gmail.com
MyPasTest: MRCS A Online - Jan Exam 2015
02. Anatomy; Thorax(120Qs)
-----------------------------------------------------------------------------------------------------------------
and (rarely) the subclavian vein can be affected, leading to a variety of upper limb, neck and upper
back symptoms. Cervical ribs, Pancoasts tumour and preganancy can all cause the condition, but
it is usually secondary to trauma or repetitive strain injuries.

71. While performing a surgical procedure in the mid-region of the thorax the surgeon accidentally
injured an important structure that lies immediately anterior to the thoracic duct. Which one of the
following structures was most likely to be injured? Single best answer question choose ONE true Page |
option only. 102
Aorta YOUR ANSWER
Azygos vein
Oesophagus CORRECT ANSWER
Superior vena cava
Trachea.

In the mid-thorax, the aorta, thoracic duct and azygos vein are all posterior to the oesophagus.
(They are in that order, from left to right.) The superior vena cava and the trachea are not located in
the mid-thorax - the superior vena cava terminates as it feeds into the right atrium and the trachea
ends as it splits into the two main stem bronchi, which enter the lungs.

72. A 48-year-old female smoker presents with acute severe chest pain and ST elevation on the
ECG. Troponin levels are raised and suggestive of myocardial infarction. She undergoes Primary
Coronary Angioplasty and is noted to have an occlusion within the short artery that passes
between the left atrial appendage and the pulmonary trunk. Which vessel has been occluded?
Conus artery YOUR ANSWER
Left circumflex artery
Left coronary artery CORRECT ANSWER
Posterior descending artery
Right coronary artery.

The left coronary artery originates from the ascending aorta just distal to the aortic valve within the
sinus of Valsalva. It courses between the pulmonary trunk and the left atrial appendage before
dividing into the circumflex and left anterior descending (LAD) arteries. In some cases, it fails to
develop, and the circumflex and LAD originate from the left aortic sinus.

73. A 32-year-old man was shot in the chest. The bullet punctured a vessel that courses across the
mediastinum in an almost horizontal fashion. Which one of the following vessels was injured?
Single best answer question choose ONE true option only.
Left subclavian artery YOUR ANSWER
Left subclavian vein
Left brachiocephalic vein CORRECT ANSWER
Left internal jugular vein
-------------------------------------------------------------------------------------------------------------------------------------------------------------------------
Dr Mohammed Shamsul Islam Khan, Medical Officer, Clinical Neuro-Surgery, National Institute of Neuro-Sciences and Hospital Sher-E-Bangla
Nagar, Dhaka-1207, Bangladesh. Mobile: +880 1713 455 662, E-mail: drsikhan@gmail.com
MyPasTest: MRCS A Online - Jan Exam 2015
02. Anatomy; Thorax(120Qs)
-----------------------------------------------------------------------------------------------------------------
Left common carotid artery.

The left brachiocephalic vein joins with the right brachiocephalic vein to form the superior vena
cava on the right side of the body. Therefore, the left brachiocephalic vein must course across the
mediastinum to reach its destination. The left subclavian artery and vein are lateral to the
mediastinum, while the left jugular and common carotid travel vertically. Page |
74. A 66-year-old man undergoes a left lung lobectomy for a tumour originating from the lingula. 103
Which lobe of the left lung contains the lingula? Select one answer only.
Inferior lobe YOUR ANSWER
Middle lobe
Superior lobe CORRECT ANSWER
Apical lobe
Basal lobe.

The left lung is divided into superior and inferior lobes by a long deep oblique fissure. The superior
lobe has a wide cardiac notch on its anterior border. The anteroinferior part of the superior lobe
also has a small tongue-like projection called the lingula.

75. A 65-year-old heavy smoker with type 2 diabetes mellitus presents to A and E with central chest
pain radiating to his back. Whilst in resus, he is noted to drop his blood pressure, and on careful
examination his JVP is increases on inspiration and his pulse fades. What is the most likely
complication of aortic dissection that has developed? Select one answer only.
Aortic regurgitation YOUR ANSWER
Cardiac tamponade CORRECT ANSWER
Myocardial ischaemia
Myocardial infarction
Right ventricular failure.

Aortic dissection results from an aortic intimal tear with blood tracking between the intima and
media to create a false lumen. In doing this the aortic wall is weakened and may rupture.
Intrapericardial rupture will cause cardiac tamponade. Additionally, as the dissection progresses,
flow down arterial branches may be compromised. Thus myocardial, cerebral and limb ischaemia
are potential consequences. Lastly, aortic dissection can lead to dilatation of the aortic root and
annulus leading to acute aortic regurgitation.

76. A 72-year-old diabetic male presents with central chest pain radiating to his neck. His ECG
shows he has developed complete heart block thought to be secondary to ischaemia affecting the
AV node. In what proportion of people is the AV node supplied by the right coronary artery? Select
one answer only.
1% YOUR ANSWER
-------------------------------------------------------------------------------------------------------------------------------------------------------------------------
Dr Mohammed Shamsul Islam Khan, Medical Officer, Clinical Neuro-Surgery, National Institute of Neuro-Sciences and Hospital Sher-E-Bangla
Nagar, Dhaka-1207, Bangladesh. Mobile: +880 1713 455 662, E-mail: drsikhan@gmail.com
MyPasTest: MRCS A Online - Jan Exam 2015
02. Anatomy; Thorax(120Qs)
-----------------------------------------------------------------------------------------------------------------
25%
50%
80% CORRECT ANSWER
95%.
Page |
The right coronary artery gives off a posterior interventricular branch. The right coronary artery 104
supplies the AV node in 80% of cases.

77. An 82-year old man undergoes insertion of central line through the right internal jugular vein.
The wire used during the procedure damages the right atrium. Which of the following is true
regarding the right atrium? Select one answer only.
It contains the sinoatrial node YOUR ANSWER (Correct)
It forms the inferior border of the heart
It has an anterior wall formed by the interatrial septum
It has the coronary sinus opening above the septal cusp of the mitral valve
It lies posterior to the left atrium.

The right atrium forms the right border of the heart, lies anterior to the left atrium and so its
posterior wall is the interatrial septum. The sinoatrial node lies near the opening of the superior
vena cava, lateral to the sulcus limitans. The coronary sinus (responsible for the venous drainage
of the myocardium) opens into the atrium above both the opening of the inferior vena cava and the
septal cusp of the tricuspid valve.

78. A 6-year-old boy accidentally aspirates a small coin. A chest X-ray shows this to be lodged in
the right main bronchus. Which of the following descriptions of the main bronchial airways is
correct? Select one answer only.
Aspiration pneumonitis is more common in the right lower lobe than the left YOUR ANSWER
(Correct)
The azygous vein crosses the left main bronchus
The left bronchus is shorter than the right
The left main bronchus divides before entering the lung
The right main bronchus has a narrower diameter than the left.

The right main bronchus is shorter (approximately 2.5 cm long), wider and runs more vertically than
the left main bronchus. The right main bronchus gives off the lobe branch (before entering the lung
hilum) and passes inferior to the pulmonary artery before entering the hilum of the lung
(approximately T5).

It is important to remember that the azygos vein arches over the right main bronchus from the
posterior aspect as it passes to the SVC. The pulmonary artery lies inferior and then anterior to it.

-------------------------------------------------------------------------------------------------------------------------------------------------------------------------
Dr Mohammed Shamsul Islam Khan, Medical Officer, Clinical Neuro-Surgery, National Institute of Neuro-Sciences and Hospital Sher-E-Bangla
Nagar, Dhaka-1207, Bangladesh. Mobile: +880 1713 455 662, E-mail: drsikhan@gmail.com
MyPasTest: MRCS A Online - Jan Exam 2015
02. Anatomy; Thorax(120Qs)
-----------------------------------------------------------------------------------------------------------------
The left main bronchus is about 5 cm long and, unlike the right, does not give off any branches
before entering the hilum of the left lung at the level of T6.

79. A patient with a cystic swelling in his left chest underwent a computer tomography-guided
biopsy. The radiologist inserted the biopsy needle into the ninth intercostal space along the mid-
axillary line to aspirate the swelling and obtain tissue for histological diagnosis. The swelling is
most likely to be in which space? Single best answer question choose ONE true option only. Page |
Cardiac notch YOUR ANSWER 105
Costodiaphragmatic recess CORRECT ANSWER
Costomediastinal recess
Cupola
Oblique pericardial sinus.

The costodiaphragmatic recess is the lowest extent of the pleural cavity or sac. It is the part of the
pleural sac where the costal pleura changes into the diaphragmatic pleura. It is also the area into
which a needle is inserted for thoracocentesis and it is found at different levels in different areas of
the thorax. At the mid-clavicular line, the costodiaphragmatic recess is between ribs 6 and 8; at the
mid-axillary line it is between ribs 8 and 10; and at the paravertebral line it is between ribs 10 and
12. Therefore, inserting the needle just below the ninth rib at the mid-axillary line should put you in
the costodiaphragmatic recess.

The cardiac notch is a structure on the left lung, which separates the lingula below from the upper
portion of the superior lobe of the left lung. The costomediastinal recess is found where the costal
pleura become the mediastinal pleura. The cupola is the part of the pleural cavity, which extends
above the level of the first rib into the root of the neck. The oblique pericardial sinus is an area of
the pericardial cavity located behind the left atrium of the heart.

80. You suspect a lower respiratory tract infection in an elderly post operative patient and request a
chest X-ray. Which of the following chest X-ray appearances is most likely? Single best answer
question choose ONE true option only.
Blunting of the costophrenic angles YOUR ANSWER
Indistinct hemidiaphragms CORRECT ANSWER
Kerley B lines
Indistinct right heart border
Indistinct left heart border.

Blunting of the costophrenic angles occurs when a (relatively) small volume of fluid has collected
in the pleural spaces. This can occur in a number of scenarios, but it is not the most likely
appearance in the above patient. Immobilisation puts such a patient at risk of basal atelectasis,
subsequent consolidation and therefore a less distinct air-tissue interface at the diaphragm on
chest X-ray.

-------------------------------------------------------------------------------------------------------------------------------------------------------------------------
Dr Mohammed Shamsul Islam Khan, Medical Officer, Clinical Neuro-Surgery, National Institute of Neuro-Sciences and Hospital Sher-E-Bangla
Nagar, Dhaka-1207, Bangladesh. Mobile: +880 1713 455 662, E-mail: drsikhan@gmail.com
MyPasTest: MRCS A Online - Jan Exam 2015
02. Anatomy; Thorax(120Qs)
-----------------------------------------------------------------------------------------------------------------
Kerley B lines occur when there is interstitial oedema. An indistinct right heart border occurs with
right middle lobe consolidation, and an indistinct left heart border occurs when there is left upper
lobe consolidation. Both of these are less likely sites for consolidation given the above history.

81. A 72-year-old hypertensive male undergoes a CABG following an angiogram whilst being
investigated for angina, which showed triple vessel disease. Which of the following should be Page |
prescribed long term to reduce further myocardial complications or development of atherosclerosis
in the bypass conduits? Select one answer only. 106
Abciximab YOUR ANSWER
Aspirin CORRECT ANSWER
Dipyridamole
Enoxaparin
Warfarin.

The overall mortality for CABG is about 58% at 5 years. As part of the long-term prevention of
further myocardial complications and development of atherosclerosis in the bypass conduits or in
the native arteries, lifelong aspirin is advocated. Warfarin is not required following CABG unless a
mechanical valve has been inserted or for persistent postoperative atrial fibrillation.

Coronary artery stenting is a useful technique for recurrent coronary artery disease.
Dipryridamole is used mainly for the secondary prevention of stroke and TIA. Interestingly it can
cause a vasodilation of healthy coronary arteries, and has no effect on narrowed coronary arteries.
This can result in a steal phenomena occurring which will worsen ischaemic areas.

82. A 67-year-old male presents with severe chest pain following a bout of prolonged vomiting.
Computerised tomography reveals oesophageal perforation with associated mediastinitis. Which
artery supplies the upper one third of the oesophagus?
Ascending pharyngeal artery YOUR ANSWER
Inferior thyroid artery CORRECT ANSWER
Lingual artery
Oesophageal artery
Superior thyroid artery.

The oesophagus is divided into three anatomical sections. The upper one third receives blood
supply from the inferior thyroid artery, the second from the oesophageal arteries, and the third from
the left gastric artery.

83. Where would you visualise the azygous lobe on an antero-posterior (A-P) chest X-ray? Single
best answer - choose ONE true option only.
Left lower zone YOUR ANSWER
Left middle zone
-------------------------------------------------------------------------------------------------------------------------------------------------------------------------
Dr Mohammed Shamsul Islam Khan, Medical Officer, Clinical Neuro-Surgery, National Institute of Neuro-Sciences and Hospital Sher-E-Bangla
Nagar, Dhaka-1207, Bangladesh. Mobile: +880 1713 455 662, E-mail: drsikhan@gmail.com
MyPasTest: MRCS A Online - Jan Exam 2015
02. Anatomy; Thorax(120Qs)
-----------------------------------------------------------------------------------------------------------------
Left upper zone
Right upper zone CORRECT ANSWER
Right lower zone.

An azygous lobe is seen in about 0.5% of routine chest X-rays and is a normal variant. It is seen as Page |
a reverse comma sign behind the medial end of the right clavicle.
107
84. A 2-day-old boy presents with regurgitation of saliva, choking after feeds and drooling. A
diagnosis of oesophageal atresia without fistula is made following further investigation. Which
foetal abnormality would you have expected to see during antenatal ultrasound screening?
Anencephaly YOUR ANSWER
Aneuploidy
Oligohydramnios
Polyhydramnios CORRECT ANSWER
Subchorionic haemorrhage.
Oesophageal atresia occurs in 1 in every 3,000-4,500 live births. In cases of atresia without fistulae,
nearly all mothers will have demonstrable polyhydramnios on ultrasound screening.

85. Endoderm is one of the germ layers formed during embryogenesis. Which of the following
organs is a derivative of the endoderm? Single best answer question choose ONE true option
only.
Adrenal medulla YOUR ANSWER
Dermis of the skin
Epithelial part of the tympanic cavity CORRECT ANSWER
Gonads
Lens.

Derivatives of endoderm include the epithelium of the gastrointestinal tract and its associated
glands as well as glandular cells of the liver and pancreas, epithelium of the urachus and urinary
bladder, epithelium of respiratory passages (the pharynx, trachea, bronchi and alveoli), epithelial
parts of the tonsils, thyroid, parathyroids, tympanic cavity and thymus and epithelial parts of the
anterior pituitary.

86. In this case, which region of myocardium would you expect to be most affected? Single best
answer - choose ONE true option only.
The right atrium YOUR ANSWER
The right ventricle
The anterior septum
The anterior left ventricular wall
-------------------------------------------------------------------------------------------------------------------------------------------------------------------------
Dr Mohammed Shamsul Islam Khan, Medical Officer, Clinical Neuro-Surgery, National Institute of Neuro-Sciences and Hospital Sher-E-Bangla
Nagar, Dhaka-1207, Bangladesh. Mobile: +880 1713 455 662, E-mail: drsikhan@gmail.com
MyPasTest: MRCS A Online - Jan Exam 2015
02. Anatomy; Thorax(120Qs)
-----------------------------------------------------------------------------------------------------------------
The posterior portion interventricular septum and the posterior left ventricular wall CORRECT
ANSWER.

The coronary system consists of left and right coronary arteries, which arise immediately above the
aortic valve. They are unique in that they fill during diastole, when not occluded by valve cusps and
when not squeezed by myocardial contraction. Page |
The right coronary artery arises from the right coronary sinus, giving off branches supplying the 108
right atrium and right ventricle. It then continues as the posterior descending coronary artery,
which supplies the posterior portion of the interventricular septum and the posterior left ventricular
wall.
The left coronary artery divides into the left anterior descending (LAD) and circumflex arteries. The
LAD runs in the anterior interventricular groove and supplies the anterior septum and the anterior
left ventricular wall. The left circumflex artery gives off branches that supply the left atrium and left
ventricle.
The sinus node is supplied by the right coronary artery in around 60% of people, the AV node in
around 90%.
87. A 29-year-old woman undergoes an emergency thoracotomy following a gunshot wound to the
left side of the chest. On exposing the left main bronchus, which of the following structures may be
encountered on its anterior surface? Select one answer only.
Aorta YOUR ANSWER
Azygos vein
Phrenic nerve CORRECT ANSWER
Recurrent laryngeal nerve
Vagus nerve.
The left main bronchus passes downwards and outwards below the aortic arch, anterior to the
descending aorta and oesophagus. The pulmonary artery loops over it, the vagus nerve lies just
posterior, the phrenic nerve in front, and the hemiazygos vein is posterior to the aorta.

88. A hypertensive, heavy smoking, 73-year-old man suffers a massive cardiac infarct following
occlusion of his anterior interventricular artery, (anterior descending artery). Angiography is
performed to demonstrate the coronary vessels. Which anatomical relationship of these vessels
should be borne in mind? Single best answer question choose ONE true option only.
The anterior interventricular artery arises above the left posterior aortic cusp YOUR ANSWER
The anterior interventricular artery supplies almost all of the left ventricle CORRECT ANSWER
There is a rich collateral circulation between the right and left coronary arteries
The circumflex artery is the major branch of the right coronary artery
The posterior interventricular artery is a branch of the circumflex artery.

-------------------------------------------------------------------------------------------------------------------------------------------------------------------------
Dr Mohammed Shamsul Islam Khan, Medical Officer, Clinical Neuro-Surgery, National Institute of Neuro-Sciences and Hospital Sher-E-Bangla
Nagar, Dhaka-1207, Bangladesh. Mobile: +880 1713 455 662, E-mail: drsikhan@gmail.com
MyPasTest: MRCS A Online - Jan Exam 2015
02. Anatomy; Thorax(120Qs)
-----------------------------------------------------------------------------------------------------------------
The left coronary artery arises above the left posterior aortic cusp. After a short course it divides
into an anterior interventricular and a circumflex branch. The former is the main arterial supply to
the left ventricle. Unfortunately, there is only a poor collateral supply between the branches of the
two coronary arteries. The posterior interventricular artery arises from the right coronary artery.

89. A 39-year-old woman undergoes a transthoracic echo following the recent onset of heart failure Page |
with cyanosis. This confirms an Atrial Septal Defect (ASD) and it is thought she has Eisenmenger
syndrome. The foramen ovale normally closes at birth to form the oval fossa. Which of the 109
following best describes the location of the oval fossa within the right atrium? Select one answer
only.
Anterolateral wall YOUR ANSWER
Anteromedial wall
Inferior aspect
Posterolateral wall
Posteromedial wall CORRECT ANSWER.

The two parts of the right atrium are separated externally by a groove on the posterior aspect of the
atrium known as the sulcus terminalis and internally by the crista terminalis, which extends
between the two vena caval orifices. The fossa ovalis is found on the interatrial septum, which
forms the posteromedial wall of the right atrium. The opening of the coronary sinus is guarded by a
semicircular valve that closes the orifice during contraction of the right atrium.

90. A 35-year-old scaffolder falls from a height of ten metres and complains of back pain. On
examination, he is unable to flex his knee, but can extend it following passive flexion. At which
vertebral level has there been an injury to the spinal cord?
L1 YOUR ANSWER
L3
L5 CORRECT ANSWER
S2
T12.

Quadriceps (knee extenders) includes rectus femoris, vastus lateralis, vastus medialis and vastus
intermedius. These muscles are innervated by fibres originating from the posterior division of
ventral rami of the femoral nerve (L2, 3, 4). The hamstrings (biceps femoris, semitendinosus and
semimembranosus) are supplied by tibial component of sciatic nerve (L5, S1) and flex the knee
joint.

91. Which of the following structures is located in the anterior mediastinum on computed
tomography (CT)? Single best answer question choose ONE true option only.
Thymus YOUR ANSWER

-------------------------------------------------------------------------------------------------------------------------------------------------------------------------
Dr Mohammed Shamsul Islam Khan, Medical Officer, Clinical Neuro-Surgery, National Institute of Neuro-Sciences and Hospital Sher-E-Bangla
Nagar, Dhaka-1207, Bangladesh. Mobile: +880 1713 455 662, E-mail: drsikhan@gmail.com
MyPasTest: MRCS A Online - Jan Exam 2015
02. Anatomy; Thorax(120Qs)
-----------------------------------------------------------------------------------------------------------------
Oesophagus
Aorta
Heart
Trachea.
Page |
The anterior mediastinum is bordered anteriorly by the sternum and posteriorly by the great 110
vessels. It contains the thymus, lymph nodes, fat, and vessels. Disorders of the anterior
mediastinum are generally thymic, thyroid (substernal goitre), teratoma (and other germ cell
tumors), and lymphomas (Hodgkin's disease, non-Hodgkin's lymphoma).

92. A 43-year-old woman undergoes parathyroid surgery. Immediately following the surgery she is
noted to have developed a hoarse voice. Which of the following is the most accurate description of
the recurrent laryngeal nerve? Select one answer only.
It lies deep to the inferior thyroid artery YOUR ANSWER
It runs between the oesophagus and trachea in the neck CORRECT ANSWER
It should be retracted during tracheostomy to avoid damage
It supplies the cricothyroid muscle
It supplies the mucous surface of the vocal cords.

The recurrent laryngeal nerve is branch of the vagus nerve. It supplies all the intrinsic muscles of
the larynx except the cricothyroid and is sensory inferior to the vocal folds. In the neck the
recurrent laryngeal nerves on both sides follow the same course, ascending in the tracheo-
oesophageal groove. As the nerve passes the lateral lobe of the thyroid it is closely related to the
inferior thyroid artery. The superior laryngeal nerve supplies the vocal cord mucosa. Damage to
one recurrent laryngeal nerve leads to a hoarse voice. Damage to both nerves can lead to aphonia.

93. A 56-year-old male presents to A&E with severe vomiting and chest discomfort. His chest X-ray
shows air in the mediastinum. Which of the following disease processes may cause this
appearance? Select one answer only.
Aortic rupture YOUR ANSWER
Aortic dissection
Cardiac tamponade
Oesophageal perforation CORRECT ANSWER
Pericarditis.

Pneumomediastinum is the presence of air in the mediastinal tissues and can be readily seen on a
chest radiograph. It is a hallmark of oesophageal perforation and large airway (trachea or bronchus)
injury, and therefore must be taken seriously. Air may track into the mediastinum via the diaphragm
following perforation of an intra-abdominal viscus. Isolated aortic rupture may cause mediastinal
widening, but will not present as pneumomediastinum. Similarly, pericarditis does not cause
pneumomediastinum.
-------------------------------------------------------------------------------------------------------------------------------------------------------------------------
Dr Mohammed Shamsul Islam Khan, Medical Officer, Clinical Neuro-Surgery, National Institute of Neuro-Sciences and Hospital Sher-E-Bangla
Nagar, Dhaka-1207, Bangladesh. Mobile: +880 1713 455 662, E-mail: drsikhan@gmail.com
MyPasTest: MRCS A Online - Jan Exam 2015
02. Anatomy; Thorax(120Qs)
-----------------------------------------------------------------------------------------------------------------

94. A 19-year old woman is shot in the chest at the level of T5. Which of the following structures is
most likely to be injured at this level? Select one answer only.
The thoracic duct as it crosses the midline YOUR ANSWER (Correct)
The bifurcation of the trachea (the carina)
Page |
The commencement of the aortic arch
111
The junction of the azygous vein as it enters the superior vena cava
T4 vertebral body.

The thoracic duct crosses the midline at T5.

The manubriosternal joint corresponds to the T4 vertebral level. At this level the following
structures are found:
- bifurcation of the trachea
- start of the aortic arch
- azygous vein entering the superior vena cava
- the left recurrent laryngeal nerve loops round the ligamentum arteriosum.

95. The aortic arch and the junction of the azygos vein with the superior vena cava are positioned
on the same plane as which spinal column structure?
T3 vertebrae YOUR ANSWER
T4 vertebrae
T5 vertebrae
T3/4 intervertebral disc
T4/5 intervertebral disc CORRECT ANSWER.

The manubriosternal joint marks the thoracic plane, which is at the level of the T4/5 intervertebral
disc. Other structures found on this plane include the aortic arch, the junction of the azygos vein
with the SVC, the 2nd costal cartilages, the tracheal bifurcation, the ligamentum arteriosum and the
decussation of the thoracic duct. The manubriosternal joint is also known as the Angle of Louis.

96. A 45-year-old man is undergoing decompression of thoracic outlet syndrome. During this, the
thoracic duct is thought to be injured and a drain is placed. Where does the thoracic duct normally
drain into? Select one answer only.
Azygos vein YOUR ANSWER
Left brachiocephalic vein CORRECT ANSWER
Left external jugular vein
Right brachiocephalic vein
Superior vena cava.

-------------------------------------------------------------------------------------------------------------------------------------------------------------------------
Dr Mohammed Shamsul Islam Khan, Medical Officer, Clinical Neuro-Surgery, National Institute of Neuro-Sciences and Hospital Sher-E-Bangla
Nagar, Dhaka-1207, Bangladesh. Mobile: +880 1713 455 662, E-mail: drsikhan@gmail.com
MyPasTest: MRCS A Online - Jan Exam 2015
02. Anatomy; Thorax(120Qs)
-----------------------------------------------------------------------------------------------------------------
The left brachiocephalic vein drains blood from: the cervical vertebrae via both vertebral veins; the
thyroid gland by the inferior thyroid veins; the first left intercostal space via the left superior
intercostal veins; and all the anterior intercostal spaces by the anterior intercostal veins draining
into the internal thoracic veins. The thoracic duct enters the vein at its commencement behind the
left sternoclavicular joint. The bronchial veins drain into the azygos/hemiazygos systems.
Page |
97. An SHO has been asked to aspirate some pleural fluid for culture and sensitivity from the left 112
pleural space of a 65-year-old man who has postpneumonic effusion. If the SHO wants to aspirate
the fluid with the patient sitting up in bed, where would the fluid tend to accumulate? Single best
answer question choose ONE true option only.
Costodiaphragmatic recess YOUR ANSWER (Correct)
Costomediastinal recess
Cupola
Hilar reflection
Middle mediastinum.

The costodiaphragmatic recess is the lowest extent of the pleural cavity or sac. It is the part of the
pleural sac where the costal pleura changes into the diaphragmatic pleura. Because this is the
inferior part of the pleural sac, fluid in the pleural sac will fall to this region when a patient sits up.
The costodiaphragmatic recess is also the area into which a needle is inserted for thoracocentesis
and it is found at different levels at different areas of the thorax.

At the mid-clavicular line, the costodiaphragmatic recess is between ribs 6 and 8; at the mid-axillary
line it is between ribs 8 and 10; and at the paravertebral line it is between ribs 10 and 12. The
costomediastinal recess is found where the costal pleura become the mediastinal pleura. The
cupola is the part of the pleural cavity that extends above the level of the first rib into the root of the
neck. The hilar reflection is the point at the root of the lung where the mediastinal pleura is reflected
and becomes continuous with the visceral pleura. Finally, the middle mediastinum is the space in
the mediastinum that contains the heart, pericardium, great vessels and bronchi (at the roots of the
lung).

98. The human embryonic heart rate is nearly twice that of the adult. Which of the following
statements regarding the development of the human heart is CORRECT? Single best answer
question choose ONE true option only.
In the second week, the endocardial tubes begin to fuse to form a single tube YOUR ANSWER
The septum primum appears in the eighth week
The heart begins to beat in the fourth week CORRECT ANSWER
The primordium of the heart forms in the cardiogenic plate located at the caudal end of the embryo
The bulboventricular loop is formed in the sixth week.

The primordium of the heart forms in the cardiogenic plate located at the cranial end of the embryo.
Angiogenic cell clusters, which lie in a horseshoe-shaped configuration in the plate, coalesce to
-------------------------------------------------------------------------------------------------------------------------------------------------------------------------
Dr Mohammed Shamsul Islam Khan, Medical Officer, Clinical Neuro-Surgery, National Institute of Neuro-Sciences and Hospital Sher-E-Bangla
Nagar, Dhaka-1207, Bangladesh. Mobile: +880 1713 455 662, E-mail: drsikhan@gmail.com
MyPasTest: MRCS A Online - Jan Exam 2015
02. Anatomy; Thorax(120Qs)
-----------------------------------------------------------------------------------------------------------------
form two endocardial tubes. These tubes are then forced into the thoracic region due to cephalic
and lateral foldings, where they fuse together forming a single endocardial tube during the third
week.

The heart begins to beat in the fourth week at about the same time that the septum primum appears
and the bulboventricular loop is formed. From the fourth week onwards, septa begin to grow in the
atria, ventricle and bulbus cordis to form right and left atria, right and left ventricles and the two Page |
great vessels - the pulmonary artery and the aorta. By the end of the eighth week, partitioning is 113
completed and the fetal heart has formed.

99. A 46-year old man is found to have a suspicious lesion in the left lower lung lobe on a CT scan.
He undergoes bronchoscopy in order to obtain a histological sample. Which of the following
statements best describes the anatomy of the left lung? Select one answer only.
The left lung has ten bronchopulmonary segments YOUR ANSWER (CORRECT)
The left lung has three bronchial openings
The left lung has three pulmonary veins
The left lung has three lobes
The left lung receives its an arterial supply from the left pulmonary artery.

The left lung has two lobes, in contrast to the right, which has three lobes. The left lung has two
bronchial openings and ten bronchopulmonary segments. Two pulmonary veins arise from the left
lung, one above and one below the oblique fissure. Note that the arterial supply to the lung
parenchyma itself originates from the bronchial arteries which arise from the aorta. The pulmonary
arteries and veins transport deoxygenated and oxygenated blood from and to the heart,
respectively. They are not the primary arterial supply of the lung parenchyma.

100. An 87-year-old female recovering from a stroke is noted to be increasingly short of breath and
confused. On examination, anteriorly her chest is clear but posteriorly harsh bronchial breathing is
detected in the lower and mid zone bilaterally. Pneumonia is diagnosed. Your consultant questions
you about the anatomy of the lungs. Select one correct answer only?
The right lung has two lobes separated by oblique fissure YOUR ANSWER
The right lung has three lobes separated by oblique and horizontal fissures CORRECT ANSWER
Regarding the lung roots, the pulmonary artery lies superiorly and the pulmonary vein lies inferiorly
whilst the bronchus lies anteriorly
The anterior margin of the left lung is straight, unlike that of the right lung
The lung is composed of approximately 300 thousand alveoli each around 0.3mm in diameter.

The right lung has three lobes separated by oblique and horizontal fissures, whilst the left lung has
two, separated by oblique fissure.

Lung roots pulmonary artery lies superiorly. Bronchus lies posteriorly and Pulmonary veins lie
inferiorly.

-------------------------------------------------------------------------------------------------------------------------------------------------------------------------
Dr Mohammed Shamsul Islam Khan, Medical Officer, Clinical Neuro-Surgery, National Institute of Neuro-Sciences and Hospital Sher-E-Bangla
Nagar, Dhaka-1207, Bangladesh. Mobile: +880 1713 455 662, E-mail: drsikhan@gmail.com
MyPasTest: MRCS A Online - Jan Exam 2015
02. Anatomy; Thorax(120Qs)
-----------------------------------------------------------------------------------------------------------------
The right lung is larger and heavier than the left but it is also shorter and wider because the right
dome of the diaphragm is higher and the heart bulge more to the left. The anterior margin of the left
lobe has the cardiac notch. The human lung is composed of approximately 300 million alveoli each
around 0.3mm in diameter.

101. A 58-year-old man undergoes coronary angioplasty for unstable angina. He suffers an
iatrogenic rupture of the right coronary artery at the time of angioplasty. Which of the following Page |
statements is true regarding the right coronary artery? Select one answer only. 114
It does NOT supply the left atrium YOUR ANSWER
It overlies the right atrial appendage
It lies posterior to the infundibulum of the right ventricle
It originates in the posterior aortic sinus
It supplies the sinoatrial node CORRECT ANSWER.

The right coronary artery originates from the anterior aortic sinus of the aortic root. It supplies the
right atrium and ventricle and variable amounts of the left atrium and ventricle. It supplies the
sinoatrial node. The sinoatrial node artery passes backwards between the right auricle and aorta,
and forms a vascular ring around the termination of the superior vena cava.

Arteriolar anastomoses between the terminations of the right and left coronary arteries exist, but
are too few and small in calibre to compensate significantly in acute coronary artery occlusion.
Iatrogenic coronary artery rupture may require emergency placement of a pericardial drain to avoid
pericardial tamponade.

102. A CT 2 in cardiothoracic surgery performs his first midline sternotomy. During this heavy
bleeding is noted. Which of the following structures is most likely to be damaged? Select one
answer only.
Descending aorta YOUR ANSWER
Inferior vena cava
Internal thoracic artery
Right brachiocephalic vein CORRECT ANSWER
Superior epigastric artery.

Sternotomy is done to gain access to the heart, lung or the mediastinal structures. In a midline
sternotomy incision the following structures are likely to be affected: the brachiocephalic trunk, the
left common carotid artery, the left subclavian artery, both brachiocephalic veins, trachea,
oesophagus, phrenic nerves, both vagi, thoracic duct, left recurrent laryngeal nerve and thymus in
children.

The internal thoracic or the internal mammary arteries run 2 cm lateral to the lateral edge of the
sternal border and hence not affected in a midline sternotomy incision. The inferior vena cava
enters the thoracic cavity piercing the diaphragm at the level of T8 vertebra and opens into the base

-------------------------------------------------------------------------------------------------------------------------------------------------------------------------
Dr Mohammed Shamsul Islam Khan, Medical Officer, Clinical Neuro-Surgery, National Institute of Neuro-Sciences and Hospital Sher-E-Bangla
Nagar, Dhaka-1207, Bangladesh. Mobile: +880 1713 455 662, E-mail: drsikhan@gmail.com
MyPasTest: MRCS A Online - Jan Exam 2015
02. Anatomy; Thorax(120Qs)
-----------------------------------------------------------------------------------------------------------------
of the right atrium. It is not contained within the mediastinum and so not affected during a midline
sternotomy incision..

103. A 60-year-old previously fit and well man dies suddenly after two separate syncopal episodes.
At post mortem examination, which one of the following congenital cardiac defects that could be
associated with his sudden death would you expect to find? Single best answer question choose
ONE true option only. Page |
Dextrocardia with situs inversus YOUR ANSWER 115
Patent foramen ovale
Bicuspid aortic valve CORRECT ANSWER
Ventricular septal defect
Single coronary artery.

Whilst isolated dextrocardia can be associated with severe cardiac anomalies, dextrocardia with
situs inversus has a low incidence of accompanying defects and the heart functions normally.
Isolated patent foramen ovale is of no haemodynamic significance. The relatively common (1-2% of
people) bicuspid aortic valve leads to accelerated calcific stenosis, which is associated with
syncope and sudden death. Small ventricular septal defects are insignificant, whereas large defects
lead to massive left to right shunts. A single coronary artery is not common, but does occur. When
present, there is no clinical consequence unless disease (such as atheroma) affects the vessel.

104. A 23-year-old female is being considered for a thoracoscopic sympathectomy for refractory
hyperhidrosis affecting both axillae and hands. She has failed to respond to conservative and
medical therapy to date. Which of the following descriptions most accurately describes how the
sympathetic trunk passes from the abdomen to the thorax? Select one answer only.
Adjacent to the inferior vena cava and right phrenic nerve YOUR ANSWER
Adjacent to the oesophagus within the right crus of the diaphragm
Posterior to the lateral arcuate ligament over quadratuslumborum
Posterior to the medial arcuate ligament over psoas major CORRECT ANSWER
Within the vertebral canal.

The thoracic sympathetic chain lies on the heads of the ribs, anterior to the posterior intercostal
vessels, immediately under cover of the pleura, with the splanchnic nerves passing from the chain
medially and anteriorly over the vertebral bodies. The thoracic sympathetic chain receives white
rami from all the intercostal nerves, and passes into the abdomen under the medial arcuate
ligament of the diaphragm.

105. A 72-year-old male presents with tearing chest pain radiating to his back. He has a past
medical history of poorly controlled hypertension and is a heavy smoker. Imaging of his thorax
shows a Stanford type B aortic dissection. How is this type of dissection normally managed? Select
one answer only.
-------------------------------------------------------------------------------------------------------------------------------------------------------------------------
Dr Mohammed Shamsul Islam Khan, Medical Officer, Clinical Neuro-Surgery, National Institute of Neuro-Sciences and Hospital Sher-E-Bangla
Nagar, Dhaka-1207, Bangladesh. Mobile: +880 1713 455 662, E-mail: drsikhan@gmail.com
MyPasTest: MRCS A Online - Jan Exam 2015
02. Anatomy; Thorax(120Qs)
-----------------------------------------------------------------------------------------------------------------
Endovascular repair YOUR ANSWER
Hybrid procedure
Lifestyle advice
Medical treatment CORRECT ANSWER
Page |
Open surgery.
116
Aortic dissection results from an aortic intimal tear with blood tracking between the intima and
media to create a false lumen. In doing this the aortic wall is weakened and may rupture. Stanford
type A dissections involve the ascending aorta or aortic arch and are treated surgically. Stanford
type B dissections do not involve the arch or ascending aorta and are treated medically.

There is an association with Marfans syndrome. The operation involves replacing the ascending
aorta with a prosthetic graft to prevent dissection back towards the aortic valve and the coronary
ostia. The rest of the aorta is not replaced. Aortic dissection is normally seen as widened
mediastinum on a chest radiograph, but a normal chest radiograph does not exclude aortic
dissection.

106. A 45-year-old female is brought in following a trauma call, having been involved in a RTA. On
arrival she is very short of breath, and complaining of right chest pain. On examination, she is
cyanosed, has absent air entry on the right chest and is hyper-resonant on percussion. The trachea
is deviated from the mid-line. What is the immediate management? Select one answer only.
Chest drain insertion on right YOUR ANSWER
Chest CT
Chest X-ray
Needle thoracocentesis CORRECT ANSWER
Thoracotomy.

A tension pneumothorax results from blunt or penetrating trauma, with or without associated rib
fractures. A tension pneumothorax results from any lung parenchymal or bronchial injury that acts
as a one-way valve and allows atmospheric air to move into an intact pleural space but prevents the
exit of that air. As pressure within the intrapleural space increases, there is deviation of the trachea,
heart and mediastinal structures to the contralateral side.

The other signs and symptoms include: chest pain, dyspnoea or respiratory distress, cyanosis,
decreased breath sounds on the affected side, hyperresonance of the chest wall on percussion of
the affected side, distended jugular veins (due to obstruction of superior vena cava) and
hypotension. Tension pneumothorax is a clinical diagnosis and a life-threatening emergency. It is
essential to relieve the pressure as soon as possible and no time should be wasted in the
Emergency Department to acquire radiological imaging once a diagnosis of tension pneumothorax
is made.

-------------------------------------------------------------------------------------------------------------------------------------------------------------------------
Dr Mohammed Shamsul Islam Khan, Medical Officer, Clinical Neuro-Surgery, National Institute of Neuro-Sciences and Hospital Sher-E-Bangla
Nagar, Dhaka-1207, Bangladesh. Mobile: +880 1713 455 662, E-mail: drsikhan@gmail.com
MyPasTest: MRCS A Online - Jan Exam 2015
02. Anatomy; Thorax(120Qs)
-----------------------------------------------------------------------------------------------------------------
The immediate management consists of inserting a large-bore cannula (14G or 16G) (needle
thoracocentesis) through the second intercostal space in the mid-clavicular line on the affected
side; the needle should be over the top of the rib since the intercostal vessels and nerve run just
below the rib. Although this is the ideal site of insertion and highly recommended, if required, the
needle could be inserted through an intercostal space anywhere on the affected side.

The diagnosis is confirmed when there is a sudden rush of air during needle thoracocentesis. In Page |
all patients, this should be followed by a formal chest drain (tube thoracostomy) once the patient is 117
adequately resuscitated and stabilised. Tube thoracostomy is essential to prevent reaccumulation
of air within the pleural cavity.

107. A 78-year-old man with pseudobulbar palsy, lying supine in bed, aspirates one of his tablets
into his lungs while swallowing. It would be most likely to end up in which of the following
bronchopulmonary segments? Single best answer question choose ONE true option only.
Anterior segmental bronchus of the right superior lobe YOUR ANSWER
Medial segmental bronchus of the right middle lobe
Superior segmental bronchus of the right inferior lobe CORRECT ANSWER
Medial basal segmental bronchus of the left inferior lobe
Inferior segmental bronchus of the lingular.

Remember: inhaled material tends to go into the right bronchus because it is bigger and more
vertically orientated than the left! The superior segmental bronchus branches posteriorly off the
intermediate bronchus or the inferior lobe bronchus, so it is the segmental bronchus most likely to
receive the foreign bodies that enter the right main bronchus.

108. A 1-day-old child is noted to be in respiratory distress. Imaging reveals herniation of


abdominal contents through the posterior aspect of the left hemidiaphragm. Which type of hernia
does the child have?
Amyands hernia YOUR ANSWER
Bochdaleks hernia CORRECT ANSWER
Hiatus hernia
Littres hernia
Morgagnis hernia.

Bochdalek and Morgagni herniae are both types of congenital diaphragmatic herniae. The former
always occurs posteriorly (usually on the left side) and the latter anteriorly. Bochdalek herniae tend
to be associated with more serious complications than Morgagni herniae, and have a much higher
incidence.

Urgent gastroenteric decompression and endotracheal intubation is required to stabilise the patient
prior to surgery. The mnemonic 5 Bs can be used to recall differences between the two (B =
Bochdalek, big, bad, baby, back). Littres and Amyands hernia are inguinal herniae that involve a
Meckels diverticulum and an inflamed appendix, respectively.
-------------------------------------------------------------------------------------------------------------------------------------------------------------------------
Dr Mohammed Shamsul Islam Khan, Medical Officer, Clinical Neuro-Surgery, National Institute of Neuro-Sciences and Hospital Sher-E-Bangla
Nagar, Dhaka-1207, Bangladesh. Mobile: +880 1713 455 662, E-mail: drsikhan@gmail.com
MyPasTest: MRCS A Online - Jan Exam 2015
02. Anatomy; Thorax(120Qs)
-----------------------------------------------------------------------------------------------------------------

109. You are asked to assist your consultant with a pulmonary resection. During the procedure she
begins to dissect tissue posterior to the right main bronchus. Which vessel is at risk of injury?
Azygos vein YOUR ANSWER (Correct)
Hemiazygos vein
Page |
Superior vena cava
118
Thoracic aorta
Thoracic duct.

The azygos vein arches posteriorly over the right main bronchus to join the superior vena cava
adjacent to the root of the right lung. Care must also be taken to spare the vagus nerve, which lies
posterior to the right main bronchus.

110. Regarding the muscles of respiration, which one of the following statements is true? Single
best answer - select one answer only.
Quiet inspiration is predominantly due to the action of the diaphragm YOUR ANSWER (Correct)
The scalene muscles play an important role in quiet expiration
Active inspiration is caused by the internal intercostal muscles
Internal oblique muscles alone are important in active expiration
The external intercostal muscles pull the ribs medially and inferiorly during active expiration.

The predominant muscle of inspiration during quiet breathing is the diaphragm, a dome-shaped
musculofibrous septum separating the thorax from the abdominal cavity. As the diaphragm
contracts, pleural pressure drops, which lowers the alveolar pressure. This draws air into the lungs
due to the pressure gradient from the mouth to the alveoli.

Expiration during quiet breathing is a passive process. This is caused by the relaxation of the
respiratory muscles and the return of the elastic lung and chest wall to their normal resting volume.
During exertion or activity, the external intercostals help in inspiration by raising the lower ribs
superiorly and laterally, so increasing the lateral and antero-posterior diameters of the thoracic
cavity. The scalene muscles and sternocleidomastoids also help by raising and pushing out the
upper ribs and the sternum.

Active expiration is helped by the contraction of the abdominal wall muscles (internal oblique,
external oblique, transversus abdominus and rectus abdominus). It increases the intra-abdominal
pressure, which pushes up the diaphragm, so raising the pleural pressure and drives the air out of
the lungs. The internal intercostals also help in active expiration by decreasing the thoracic volume
(by pulling down medially and inferiorly).

111. A 25-year-old man was stabbed in the right supraclavicular fossa. The knife punctured the
portion of the parietal pleura that extends above the first rib. This portion of the parietal pleura is
called the: Single best answer question choose ONE true option only.
Costodiaphragmatic recess YOUR ANSWER
-------------------------------------------------------------------------------------------------------------------------------------------------------------------------
Dr Mohammed Shamsul Islam Khan, Medical Officer, Clinical Neuro-Surgery, National Institute of Neuro-Sciences and Hospital Sher-E-Bangla
Nagar, Dhaka-1207, Bangladesh. Mobile: +880 1713 455 662, E-mail: drsikhan@gmail.com
MyPasTest: MRCS A Online - Jan Exam 2015
02. Anatomy; Thorax(120Qs)
-----------------------------------------------------------------------------------------------------------------
Costomediastinal recess
Costocervical recess
Cupola CORRECT ANSWER
Endothoracic fascia.
Page |
The cupola is the cervical parietal pleuron, which extends slightly above the level of the first rib into 119
the root of the neck. The costodiaphragmatic recess is the part of the pleural sac where the costal
pleura changes into the diaphragmatic pleura. It is the lowest extent of the pleural sac. The
costomediastinal recess is found where the costal pleura become the mediastinal pleura. The
endothoracic fascia is connective tissue between the inner chest wall and the costal parietal pleura.
The costocervical recess is a made-up term.

112. An 81-year-old male presents with progressive dysphagia and weight loss. An OGD reveals a
tumour proximal to the gastro-oesophageal junction and histology confirms this is an oesophageal
adenocarcinoma. Which of the following best describes where the oesophagus passes through the
diaphragm? Select one answer only.
Posterior to the medial arcuate ligament YOUR ANSWER
Posterior to the lateral arcuate ligament
Through the central tendon
Through the muscular sling of the left crus
Through the muscular sling of the right crus CORRECT ANSWER.

The oesophagus is formed at the lower border of the cricoid cartilage. It is crossed anteriorly by the
left main bronchus, lies behind the left atrium and passes through the muscular part of the
diaphragm to the left of the central tendon, through the muscular sling of the right crus. It is
innervated, in part, by the recurrent laryngeal nerve, not the phrenic nerve.

113. A male jockey is thrown from his horse and sustains a spinal cord injury. He presents with
difficulty breathing and ultrasound scan reveals a right hemidiaphragmatic paralysis. From which
spinal roots does the affected nerve originate?
C2-3 YOUR ANSWER
C2-4
C3-5 CORRECT ANSWER
C6-8
C7-8.

The phrenic nerve has been injured and its function and origin can be recalled by the mnemonic
C3,4,5 keeps the diaphragm alive.

-------------------------------------------------------------------------------------------------------------------------------------------------------------------------
Dr Mohammed Shamsul Islam Khan, Medical Officer, Clinical Neuro-Surgery, National Institute of Neuro-Sciences and Hospital Sher-E-Bangla
Nagar, Dhaka-1207, Bangladesh. Mobile: +880 1713 455 662, E-mail: drsikhan@gmail.com
MyPasTest: MRCS A Online - Jan Exam 2015
02. Anatomy; Thorax(120Qs)
-----------------------------------------------------------------------------------------------------------------
114. A 78-year-old male undergoes an oesophagectomy for oesophageal adenocarcinoma.
Dissection around the intrathoracic oesophagus is very difficult, with significant bleeding. Which of
the following is a posterior relation of the intrathoracic oesophagus? Select one answer only.
Aortic arch YOUR ANSWER
Azygos vein
Page |
Left common carotid artery
120
Hemiazygos vein CORRECT ANSWER
Right common carotid artery.

Relations include the right and left bronchi, the trachea, the vagus nerve, pleura, the aortic arch and
descending aorta and the thoracic duct.
Within the thorax the relations of the oesophagus are:

Anterior:
Left common carotid artery
Trachea
Left main bronchus
Pericardium

Posterior:
Thoracic vertebrae
Thoracic duct
Hemiazygos vein
The descending aorta below

On the left:
Left subclavian artery
Aortic arch
Left vagus nerve and its recurrent laryngeal branch
Thoracic duct
Left pleura

On the right:
Right pleura
Azygos vein.

115. You are asked to see a 45-year-old man who is haemodynamically compromised and plan to
insert a right subclavian line. He has a body mass index (BMI) of 38, where is the correct position
for central venous cannulation? Single best answer - choose ONE true option only.
1 cm under the mid-point of the clavicle and 0.5 cm laterally YOUR ANSWER
2 cm under the mid-point of the clavicle and 1 cm laterally CORRECT ANSWER
2.5 cm under the mid-point of the clavicle and 2 cm laterally
0.5 cm under the mid-point of the clavicle and 1 cm laterally

-------------------------------------------------------------------------------------------------------------------------------------------------------------------------
Dr Mohammed Shamsul Islam Khan, Medical Officer, Clinical Neuro-Surgery, National Institute of Neuro-Sciences and Hospital Sher-E-Bangla
Nagar, Dhaka-1207, Bangladesh. Mobile: +880 1713 455 662, E-mail: drsikhan@gmail.com
MyPasTest: MRCS A Online - Jan Exam 2015
02. Anatomy; Thorax(120Qs)
-----------------------------------------------------------------------------------------------------------------
1 cm under the mid-point of the clavicle and 1 cm laterally.

In obese patients, the standard position for right subclavian central venous cannulation is 2 cm
under the mid-point of the clavicle and 1 cm laterally. In thin patients the standard position for
insertion is 1 cm under the mid-point of the clavicle and 0.5 cm laterally. Jugular vein cannulation is
now the preferred choice for central venous catheterisation, as insertion under ultrasound Page |
guidance is associated with a much lower rate of complications than subclavian insertion. The
major hazard of the subclavian approach is arterial puncture, as the artery lies close to the vein. 121
116. A previously well 61-year-old male presents to A and E with a marked right upper limb
weakness and right facial droop. He is also noted to have a dysphasia. On taking a history from his
wife it emerges he is due to attend hospital the following day for an elective inguinal hernia repair.
He is normally on warfarin for a mechanical valve, which he stopped 6 days ago on the advice of his
GP. What complication has occurred? Select one answer only.
Amaurosis fugax YOUR ANSWER
Embolic stroke CORRECT ANSWER
Haemorrhagic stroke
Prosthetic endocarditis
Subdural haematoma.

Mechanical valvular prostheses require lifelong anticoagulation to prevent emboli and valvular
obstruction with thrombus. Heparin may be used to anticoagulate should warfarin need to be
withheld for any reason. Warfarin is a teratogen, and thus mechanical valves should be avoided in
those women planning to have children.

Prosthetic valve endocarditis is notoriously difficult to treat and will often require redo valve
replacement. Thus antibiotic prophylaxis prior to instrumentation is very important in those
patients with prosthetic valves.

117. Following a difficult cervical rib excision, the surgeon is concerned about a possible injury to
the pleura. The patient reports ipsilateral chest pain worse on inspiration, is noted to have reduced
breath sounds and the trachea remains central. There is hyperresonance evident on percussion of
the ipsilateral chest wall. Which of the following complications is most likely from the history?
Select one answer only.
Chylothorax YOUR ANSWER
Haemothorax
Pleural effusion
Pneumothorax CORRECT ANSWER
Tension pneumothorax.

-------------------------------------------------------------------------------------------------------------------------------------------------------------------------
Dr Mohammed Shamsul Islam Khan, Medical Officer, Clinical Neuro-Surgery, National Institute of Neuro-Sciences and Hospital Sher-E-Bangla
Nagar, Dhaka-1207, Bangladesh. Mobile: +880 1713 455 662, E-mail: drsikhan@gmail.com
MyPasTest: MRCS A Online - Jan Exam 2015
02. Anatomy; Thorax(120Qs)
-----------------------------------------------------------------------------------------------------------------
The pleura is a serous membrane which invests the lung itself (visceral pleura) and lines the cavity
containing the lung (parietal pleura). These two parts of the membrane are continuous at the lung
hilum and enclose a small space occupied by about 510ml of fluid produced mainly by the parietal
layer. Due to the elasticity of the lungs, intrapleural pressure is usually negative during quiet
inspiration and expiration, but during forced expiration contraction of expiratory muscles raises the
pressures in the pleural spaces and within the lungs and bronchi.
Page |
118. A 30-year-old male squash player presents to the Emergency Department following a collision 122
with his opponent. He complains of severe chest pain and further investigation reveals three
fractured ribs. There is no evidence of significant intrathoracic injury and you are satisfied that his
respiratory function has not been compromised. What is the most appropriate management?
Chest strapping YOUR ANSWER
Intercostal drain
Observation in thoracic surgery HDU
Patient Controlled Analgesia and/or intercostal block CORRECT ANSWER
Rib belt.

This patient has simple rib fractures which do not require surgical intervention. Such fractures can
be extremely painful and may necessitate PCA or intercostals block analgesia or both. Once
significant thoracic injury has been excluded it is safe to manage such patients on the ward or at
home if their pain is controlled with oral analgesia. Rib belts and chest strapping are
contraindicated.

119. A young man is about to undergo bronchoscopy for suspected inhaled peanut. Where is the
most likely site for the inhaled peanut? Single best answer question choose ONE true option only.
Right middle lobe bronchus YOUR ANSWER
Left inferior lobe bronchus
Right superior lobe bronchus
Left superior lobe bronchus
Right lower lobe bronchus CORRECT ANSWER.

The right main bronchus is wider, shorter and runs more vertically than the left main bronchus.
Consequently, foreign bodies small enough to be inhaled more commonly enter the right lung. As a
result of gravity, the right lower lobe is more likely to receive such foreign bodies.

120. Following a difficult left cervical rib excision a 33-year-old female is noted to have straw
coloured fluid in her drain and the diagnosis of a chyle leak is made secondary to a thoracic duct
injury. Which of the following best describes the vertebral level through which the thoracic duct
passes through the diaphragm? Select one answer only.
T4 YOUR ANSWER
T6
T8
-------------------------------------------------------------------------------------------------------------------------------------------------------------------------
Dr Mohammed Shamsul Islam Khan, Medical Officer, Clinical Neuro-Surgery, National Institute of Neuro-Sciences and Hospital Sher-E-Bangla
Nagar, Dhaka-1207, Bangladesh. Mobile: +880 1713 455 662, E-mail: drsikhan@gmail.com
MyPasTest: MRCS A Online - Jan Exam 2015
02. Anatomy; Thorax(120Qs)
-----------------------------------------------------------------------------------------------------------------
T10
T12 CORRECT ANSWER.

The thoracic duct crosses the diaphragm at the aortic hiatus (T12) and ascends the superior and
posterior mediastinum between the descending thoracic aorta (to its left) and the azygos vein (to its
Page |
right). The thoracic duct ascends anterior to the posterior intercostal vessels and has several
valves. At the thoracic inlet, it lies to the left of the oesophagus and arches forward over the dome 123
of the left pleura, draining into the left brachiocephalic vein. The right bronchomediastinal trunk
drains into the right subclavian vein.

Courtesy:

Dr Mohammed Shamsul Islam Khan


MBBS (CMC), FCPS-II (Neuro-Surgery)
Medical Officer, Clinical Neuro-Surgery
National Institute of Neuro-Sciences and Hospital
Sher-E-Bangla Nagar, Dhaka-1207
Bangladesh.
Mobile: +880 1713 455 662
E-mail: drsikhan@gmail.com

-------------------------------------------------------------------------------------------------------------------------------------------------------------------------
Dr Mohammed Shamsul Islam Khan, Medical Officer, Clinical Neuro-Surgery, National Institute of Neuro-Sciences and Hospital Sher-E-Bangla
Nagar, Dhaka-1207, Bangladesh. Mobile: +880 1713 455 662, E-mail: drsikhan@gmail.com

Вам также может понравиться